Highlights From The Comments On College Admissions

HalTheWise discusses a factor I missed (until I sneakily edited it in, so you may have read the later version that included it):

One very powerful contributor that Scott did not mention is that in many cases schools are directly or indically intentivized to have a low admission rate. US news & world report released the first national college ranking in 1983, and donors and board members at various schools have increasingly been using national rankings performance, which directly includes low admissions rates, as a measure of how well a school is doing.

These rankings and metrics also heavily incentivize having high yield (a large fraction of students that are admitted end up attending) which for a fixed size applicant pool also encourages accepting as few people as possible. This has led to the death of safety schools, because they would rather reject a high performing student than admit them and have them not attend.

These factors might also be a driving force behind the rise of common app, since schools are trying to get as many applicants as possible, even if it hurts the quality of their pool.


kaakitwitaasota points out that consulting is an exception to the “where you go to school doesn’t matter” principle:

A lot of top firms these days won’t even look at you if you didn’t go to the “right” college. My mother did her MBA at Northeastern, and recently had lunch with an old classmate who ended up at a top consulting firm. My mother’s classmate’s résumé would end up in the trash unread these days–Northeastern isn’t considered good enough.

So while it’s probably true on the macro level that smart kids will do just fine anywhere they end up, there is a subset of extremely prestigious, extremely well-paid jobs which will not even look at you if you didn’t get into the right institution at the age of 18–which, in practice, means that the élite are chosen on the basis of who they were at the age of 14-17. When viewed in those terms, it’s completely nuts.

I’d heard this before; my impression is that a big part of consulting is having prestigious-looking people tell you what you want to hear. If what they’re actually hiring for is prestige rather than competence per se, that could make it a special case


prunesquallor on UCs:

I believe UCs are more competitive because of cost. Personally, I got into a number of strong private schools, and chose a UC because it is significantly cheaper. In the past, this would not have been as much of a factor, because college was affordable. I’m not sure if this applies to public schools in other states. The UCs are the best public schools in the country and are able to compete with high level private schools.

This makes sense, but I’m not sure exactly what the model is. UCs are cheap, so many people across the country apply to them? (but I gave data showing out-of-staters were only 14% of UC students, plus it may not be cheap for out-of-staters). UCs are cheap, so many Californians apply? Aren’t public universities always cheap for people in the state? Maybe cheap UCs mean more top-performing Californians apply to UCs instead of private out-of-state colleges?


BlindKungFuMaster writes about a factor that could explain stories of exceptional students being rejected from everywhere they apply:

It could be the case that college admissions became more random as kids apply more often and the metrics become more vague. I.e. there used to be x acceptable applicants of which 50% were somewhat randomly selected. Now there are 2x acceptable applicants of which 25% are somewhat randomly selected. Then there are just much more kids being unlucky and missing out on all their choices, though it hasn’t really been getting harder to get in. Of course individually the remedy is applying to even more colleges.

This might increase perceptions of selectivity – if one of these bright students posts their sob story, everyone else will just think that standards are very high.


A few people chimed in with concerns about the Dale and Krueger paper. rlms wrote:

From the abstract:

“We find that the return to college selectivity is sizeable for both cohorts in regression models that control for variables commonly observed by researchers, such as student high school GPA and SAT scores. However, when we adjust for unobserved student ability by controlling for the average SAT score of the colleges that students applied to, our estimates of the return to college selectivity fall substantially and are generally indistinguishable from zero.”

So college selectivity *is* significant even after controlling for student quality as measured by SAT scores. It only ceases to be significant when you also control for some vague measure of ambition as signalled by the average SAT score of all the colleges they applied to.

And reasoner cites a 2009 Overcoming Bias post finding that a previous Dale & Krueger paper on this subject was misinterpreted to say college didn’t matter when it really did matter (Marginal Revolution also covered this). The Dale & Krueger paper I posted was an update to that one that said no, really, college doesn’t seem to matter. But I haven’t had time to look at it closely myself, so this shifts my priors a little bit.

aesthesia is also doubtful:

If I remember correctly, the Dale and Kreuger paper is somewhat limited in its conclusions. Their sample was weighted toward the high end of the achievement spectrum, so it really says something like: conditional on being accepted to Harvard, there’s not much difference in lifetime earnings between actually attending Harvard and instead choosing to attend UC Berkeley. There aren’t a lot of students admitted to Harvard who instead choose to go to the University of Southern North Dakota at Hoople, so we don’t really know what happens to them.

This is a somewhat personal issue for me: I read summaries of Dale and Kreuger’s earlier work when applying for college and decided not to bother applying to selective schools, and just went to a middling state university, thinking that going somewhere more selective wouldn’t make a difference in my future life. I’m no longer confident that was the right move. I believe I would have learned more and made better and stronger connections had I gone somewhere more difficult to get into.

But eqdw’s experience bears Dale & Krueger out:

Hello. I work at a major company involved in job search, job ads, hiring, etc. And I would like to share with you something from a quarterly status update I saw the other day.

Status update presentation had a slide outlining the results of a user study we did. We surveyed employers to find out what are the most vs least important details they look at when making a hiring decision.

Out of something like 20 different options surveyed, “where the candidate went to college” was rated dead last in importance. “Formatting of resume” was rated as more important than “where they went to college” for making a hiring decision.

This would seem to confirm conclusion #6 above


Freddie de Boer wants to remind us (another point I stealthily edited into the post later on) that we really are talking about a small subset of institutions here:

I ran the numbers myself several years ago. Out of 3000+ accredited two- and four-year colleges, something like ~150 reject more students than they accept. The large majority accept almost every student who applies. And of course only half of the population will ever enroll in college and only a third will ever finish. The people who say this is a niche problem are correct.


Gossage Vardebian on money:

Certainly the idea of getting into the best school you can, is a further manifestation of the competitiveness discussed above. But that requires another factor, the “and damn the expenses” factor. I went to State U because I wasn’t good enough to get into U of Ivy, but also because I couldn’t afford it. Well, hell, the me of 1980 couldn’t afford 2019 State U either, so hey, might as well go for Harvard too. And if my parents has enough money for State U, well then Harvard is that much less financially onerous, isn’t it? This is also a reason why more kids these days apply to out-of-state State universities, which in turn are admitting more out-of-state kids – the difference in cost is not as large as in the past. It used to be that the idea of going into tens of thousands of dollars of debt for college was insane – literally nobody I know did that. People would do it for med school or law school, and that’s it. I’m sure there were examples from other demographics where that happened, but now of course it is commonplace, as a quick Google search will confirm. Part of this is the whole “you must get into the best school possible” mindset, but part is just a “that’s just the way it is now” resignation. The mental leap to take on that kind of debt has been conditioned into families now.

Anthony adds:

The debt thing is a big change from when I went to college, but it’s driven by policy more than by parents.

The government encouraged student loans because those would get paid back while subsidizing tuition at the student end or the college end wouldn’t (directly), and since they were paid back, the had very little budget cost. With a relatively significant college premium, people were willing to take on fairly large amounts of debt (and were not very likely to default). This allowed colleges to spend more, driving up the sticker price, making loans more necessary for students who weren’t poor enough to get direct subsidies or rich enough to afford the higher price.

Unfortunately, this is a politically very difficult problem to unwind. Even though the best policy would be to stop subsidizing student loans entirely and make them dischargeable in bankruptcy, which would hugely limit the availability of student loans, that’s never going to happen.


Peffern relates their own experience:

I think point 3.4 is the most important, based on anecdotal evidence.

I’m in undergrad at a top school right now.

In high school, I was a good student – perfect SAT, good APs, reasonable GPA, etc. Despite having good accomplishments here it didn’t feel particularly effortful – I’m good at math and can structure a coherent argument, so taking the SAT was mostly just getting a good night’s sleep and studying vocabulary for a week.

I also did a lot of extracurriculars, and the work and stress load from those absolutely destroyed me. I’m not even talking about “starting homework at 11pm” kind of workload, I mean the social aspect, the cutthroat politics, the status games, and the showmanship. It’s not that I think those things are necessarily bad in and of themselves, but they’re infinitely more difficult than classwork. I’m incredibly busy these days with class and I’m not even on the same order of magnitude of stress as I was in HS.

High school students are vicious bastards. When you take the AP calculus exam, or the SAT, or even just the final for some class you take, you are only really competing against the teacher, the test, and yourself. When you do extracurriculars, you are competing against horrible entitled jerks with rich parents who make your life miserable. I would take a hundred AP exams before doing another pointless extra curricular.

I don’t know if it explains the college enrollment statistics but it certainly explains the outrage, pessimism, and anger of people my age over the process. I spent what was for past generations an exciting and important time of my life locked in a box sanding off all aspects of myself that didn’t perfectly resembke an ivy league student just to get beaten out by some kid whose father got him an internship at somewhere prestigious. That does things to people.


meh finds reason for doubting my thoughts on the Common App:

The Pew article you link has a different conclusion about the Common App:

“The expansion of the Common Application, which makes it easier for students to apply to multiple schools, doesn’t appear to be behind the increase in application volume. The Common App, as it’s called, is accepted by nearly 800 colleges and universities in the United States and several dozen overseas. Of the 1,364 institutions in our sample, 729 accept the Common App along with (or in some cases instead of) their own application forms; the other 635 use their own forms. Although one might suspect that the ease of applying to multiple schools via the Common App would result in stronger growth in application volume among those schools, there was almost no difference in 2002-2017 growth rates between the schools that used the Common App and those that didn’t.”

But adds:

But is their reasoning sound? Isn’t it possible the Common App increases applications uniformly among all schools, not just ones using the Common App?

Consider if pre-Common App, I am willing to fill out 10 applications, so I apply to 10 schools. Now with the Common App, say half of them accept it, so I only need to fill out 6 applications. I am still willing to fill out 4 more applications, so I look at my 11th choice. If they take the Common App, I apply to them for free. If not, I fill out an application, and am still willing to fill out 3 more applications. Does this lead to a similar increase in applications for both Common App and non Common App schools?

And alexhutcheson has a more complicated model for why applications per student might have increased:

I think this analysis is excellent, but I don’t think the Common App explains the increase in applications-per-admission. There’s another significant factor that causes students to apply to dozens of schools: No one can accurately forecast what a given school will cost anymore.

At some point (I believe in the late ’00s, but could be wrong), elite schools started to extend their financial aid programs to include students from middle-class families. Here’s a press release from Harvard in 2007. Yale and Princeton followed quickly, and most other elite schools seem to have done similar things, albeit with more constrained resources.

Prior to this change, a student from a family with middle-class income or above could know with reasonable certainty what a given college would cost: they would expect to pay the tuition, fees, etc. listed on the brochures. After this change, they would have no idea until after they were admitted. They might get a generous financial aid package that brings the cost down to the price of their local state university, or they might get nothing except loans. The systems used to determine financial aid packages are opaque and not well-publicized, so the outcome is unpredictable.

In 2009, I applied to a broad sample of 14 schools on the east coast. They were a mix of “elite” private and flagship public universities. My parents were comfortably middle-class. I was lucky enough to get into most of them, and so I had the opportunity to compare financial offers. In maybe 1 case out of 12, I would have had to pay the full sticker price. The rest would have been heavily “discounted”, but the discount varied widely between schools, from ~5% off the total cost of attendance, to 60% off, to 100% off (full ride). For the private universities the “discount” came from a mix of “need-based aid” and merit scholarships, while for the public universities it was exclusively from merit scholarships.

In this system, you don’t know what the financial offer is until you get in, there are possible windfalls from getting a generous financial offer, and it’s difficult to predict in advance what the financial offer will be until you get in. The incentives here are obvious: student who are conscious about the cost of their education have a strong incentive to “play the lottery” by applying to as many schools as is feasible for them, while biasing towards schools that are known to provide generous financial packages.


faoiseam says that APs might be a good objective way to track the increase in competition over time:

The College Board has lots of data on AP exams since 1996. They track how many people take exams, in what grade, by race and result, which is enough to get a sense of how people did. They also keep track of AP scholars. In 2002, 1738 people were National AP scholars, (8 APs with score 4 or more) enough to get you into HYPSM. This increased to 7k by 2005, 16k by 2010 and now is 35k, not enough to get you into a top 20 school. By this measure, college entry has got more competitive.

The number of schools offering APs has increased in that period, from 10k in 2000, to 17k in 2010, to 20k now. Looking at individual subjects, the two most taken are AP Lit, and AP Calc. 5s in lit increased 10 fold from 2000 (6978/105k) to now (61k/580k). For AP Calc BC in 2000 we have 5k 5s from 15k exams, in 2010 39k 5s, from 79k exams, and now we have 56k 5s from 139k exams. As these are the most common exams, this shows that while the number of schools barely changed in the last 10 years, the number of 5s has doubled.

In general, things have gotten twice as hard since 2010 (as measured by AP exams), and 5 times 3005, and 10 to 20 times since 2000. The AP curriculum has stayed pretty stable, so that does not explain the change. The number of schools has been pretty steady since 2010, though it did double since 2000. This does not explain all of a factor of 2, as the 10k schools that offered APs in 2000 were the better schools anyway.

This makes me question why the US does not used AP exams for college admission. If, as I estimate, the number of people who get n 5s doubles as n increases by 1, this suggests that four 4s and four 5s should be enough to get into HYPSM. The UCs should requires 8 4s. or 7 4s and 1 5, etc.

This could be done by computer when AP results come out, delaying admission until the Summer of senior year. It would be really nice to have a clear bar that was required for entry, as opposed to the current opaque system.


Steve Sailor fills in a hole in my data:

I wrote a column back in 2013 documenting that famous colleges were not expanding their freshmen class sizes at anywhere close to population growth over the last generation:

Consider the growth rate of Harvard, the world’s richest university. The number of undergraduates in its class of 1986 was 1,722. After a quarter of a century, during which the US population grew by 75 million, Harvard’s class of 2011 was 1,726: an increase of four.

This is not to say that Harvard isn’t expanding: Faculty and grad students are up, and non-teaching staff skyrocketed.

Similarly, Yale’s undergraduate student body has been the same size since 1978. Five years ago, the second-richest college announced a proposal for adding a couple of dormitories, but construction won’t proceed until another $300 million is raised.

In 2010, MIT unveiled plans to expand undergrad enrollment by six percent, which would only get it back to where it was in the 1990s.

Among the most prominent colleges, Princeton is the only one over the last generation to have actually succeeded in boosting enrollment (and that by only about ten percent) after it opened the Whitman residential college in 2007.

…and gives some fictional evidence:

Robert Heinlein’s 1950s juvenile novels are full of scenes where people ask the hero what he’s going to do for college and he says, “I dunno, I guess the day after Labor Day I’ll drive down to State and sign up for some classes.” (That was pretty much what UCLA was like when Heinlein went there for awhile in the 1930s.) But in the end of “Have Space Suit, Will Travel,” the hero gets accepted into MIT when the head of the CIA, or whatever, pulls some strings.

This concept of heading down to the college the day it opens to sign up for classes reminds me of an anecdote I rejected for the main post because it was too unbelievable. Wikipedia quotes Supreme Court Justice Byron White saying he went to Yale for law school because he was planning to go to Harvard, but got sick enough on the train to Boston that he decided to get off at New Haven and go to Yale instead. But I haven’t been able to find any source better than Wikipedia for this, so I don’t know if it’s true. Does it at least mean that this was plausible enough in the 1930s (when White was going to college) to make a believable urban legend?


graeme on HackerNews gives the Canadian perspective:

You may be interested in the view from up North. Joseph Heath, a Canadian academic has commented on how the top three Canadian schools teach more students than the top ten American schools. I’ll excerpt:

————-

“But of course there’s a reason that it’s so difficult to get into Yale – it’s because Yale has only 5,400 students, in a country of over 315 million people! By contrast, McGill has over 30,000, and University of Toronto has 67,000 undergraduates, serving a country of only 35 million people. That means there’s roughly one spot at Yale for every 58,000 Americans, compared to one spot at McGill for every 1,100 Canadians. No wonder American life is more competitive.

Furthermore, all of the best schools in the United States are tiny. Here is a list of the top 10, as ranked by U.S. News & World Report, along with the number of students (undergraduate, I believe):

Princeton: 5,336

Harvard: 6,658

Yale: 5,405

Columbia: 6,068

Stanford: 7,063

Chicago: 5,590

Duke: 6,655

MIT: 4,503

Upenn: 9,682

CIT: 997

Dartmouth: 4,193

That means the top 10 universities in the United States – a country of over 315 million people – at any given time are educating a grand total of only 62,150 students.

By contrast, here are the rough numbers of undergraduates at the top 3 Canadian universities:

McGill: 30,000

UBC: 47,500

UofT: 67,000

So the top 3 Canadian schools are at any given time educating a grand total of 144,500 students – more than twice the total of the top 10 U.S. schools. (In fact, the University of Toronto alone has more student capacity than the top 10 U.S. schools combined.) The United States has almost exactly 9 times the population of Canada, so in order to have the same sort of capacity in higher education, the top 27 schools in the United States would have to have 1.3 million students.”

This suggests a model where people only have so many cognitive real estate devoted to remembering what the good schools are, and so if your top three schools are very large, you can have many people in the “cognitive elite”, but if your top three schools are small the elite will seem more selective. Seems like a stupid way to run an educational system 🙁


jumpinjacksplash offers a model of how an arms race would work:

I think what people may be missing is that time is a thing:

In a world where there are 10 places at Harvard every year but 9 applicants, everyone who can read Latin gets in, and no-one bothers to study harder than what it takes them to meet the minimum requirement.

In a world where there are 10 places at Harvard but 11 applicants, the worst Latin-reader doesn’t get in. But someone who realises they’re the worst Latin readers will practice more, and someone else will be the worst. That person will then practice enough to make someone else be the worse, who then practices a bit more. Given enough time, the standard keeps rising as everybody needs to stay ahead of everybody else; eventually everyone hits the ceiling of Latin literacy and starts taking up the violin, then competes on violin-skills until they all have to start climbing Everest.

The only equilibrium is where everyone exerts maximum effort on top of their abilities. It takes a long time for this to become even remotely necessary (I still don’t think we’re close to being there). But a world where the first 18 years of your life consists of maximum effort to get into college is a dystopian nightmare, hence as we trend towards it everything gets constantly worse.

This also explains the anecdotal people who expect to swim into Berkeley but get rejected from UCLA: their parents think they’ve massively overshot what’s required, but they’ve failed to realise how far down the slope we’ve slid.


I failed in my original goal for this piece, which was to present an account of competition getting tougher across society. There wasn’t enough data, so I fell victim to a streetlight effect where I concentrated on colleges, and then another streetlight effect where I concentrated on Harvard (which is the best-documented college). But the reasons why competition is getting tougher at Harvard probably don’t generalize, so I ruined my chance to have a really good post about modern competition. Still, a few people stuck to the original spirit and discussed increasing competition as it was showing itself in their own field.

anechoicmedia from the subreddit on TV:

This reminds me of a depressing perspective relayed by Brady Haran on podcast. An acquaintance had described getting started in the TV business back in what would probably have been the 1980s or 90s. Like most young people of the pre-computer world he arrived with no independent film making experience. As he describes it, he presented himself to a major TV station, applied for a job, and was accepted into an apprenticeship of sorts, in which they taught him all the ways of TV-news-making from the ground up, which he then went on to excel at.

Brady, himself a BBC veteran, remarked that such a thing could never happen today. Entry positions are ferociously competitive. Young adults applying today can be expected to have had a quality camera in their hands since childhood. There are no apprenticeships; New hires are expected to show up with fundamental knowledge already in hand.

But more than that, the modern ability to signal your ~passion~ via independent YouTube/Vimeo presence means you are really expected to show up with an entire history of volunteer video production going back years. How can you be worthy of working in the BBC if you haven’t been using the tools at your disposal to produce content for years already? You must not really want it — you’d be laughed out of the office without an extensive list of “extracurriculars” in hand. The ladder of opportunity had been yanked quite a few rungs upward.

Something like this may also be at work with the expectation that “real programmers”, especially younger ones, have many side projects, open source contributions, etc.

And jaghataikhan on consulting again:

This reminds me of my field (management consulting). It’s pretty competitive to get into the big three (McKinsey/Bain/BCG), so the level of preparation in applicants is also a runaway arms race. One of my buddies who went to MIT said he’d known people who’d prepped for case interviews for two years!

In contrast, I remember seeing some pre Recession recruiting videos at my firm where there was a girl from Yale saying something to the effect of “I didn’t even know what management consulting was before I interviewed!”

I can’t tell you how quaint/laughable that sounds vs. the extreme levels of competition for the same jobs today. Basically if you don’t have a top resume/school/marks/extracurrics/internships/cases/etc from day 1 (and still a matter of luck btw unless your dad is a partner or something), don’t even bother applying. And I’m certain the level of competition is skyrocketing daily.

KingWalrax gives the only thing I’ve heard that could possibly be a general explanation of the phenomena, though I don’t know if it’s right:

Ex-YCombinator President Sam Altman wrote a decent essay explaining how Zero-Sum Existential Competition & Conflict arise from a lack of baseline Growth in any given system. Whatever you think of Tech/YC, it’s a good read and Sam is a smart guy and he’s on-the-money here.

GDP Growth is always spiky/noisy, and highs average out with lows, making it difficult to discuss overall performance. But Sam’s graphs tell a pretty decent picture: we haven’t had a year of 10% GDP Growth since the 1980s, but our recession-year performance (i.e. 2008) is as bad as ever.

Your essay here already tied the broader economic landscape in to College a few times (Farming & Sources of employment in pre-1800s New England). It strikes me as relevant again here in this post-WW2 era.

Growth declined. Therefore: Zero-Sum Competition increased. The only other option was individual actors settled for lower individual outcomes, and you wrote the best essay out there on why that doesn’t happen.

I’ve seen conflicting data on whether long-term growth has really decreased and when that started, and you can see some discussion of this in the comments. If it has, it would explain a lot, and would mean there’s still room for things to get worse.

This entry was posted in Uncategorized and tagged . Bookmark the permalink.

224 Responses to Highlights From The Comments On College Admissions

  1. Murali says:

    Another anecdote.

    Edmund Gettier got tenure with just one paper: Is Justified True Belief Knowledge? And he hasn’t published anything else since. It may very well be the most cited paper in epistemology, but he is still slacking. Nowadays you couldn’t even get an assistant professorship with just one paper.

    • AnthonyC says:

      It would be very unlikely you’d even be able to finish grad school with just one paper, I think.

      • Eli says:

        I was told just this week that submitting two (full) papers in the first year of my PhD was “a running start”, and I’m of course working on another paper right now. Finish grad school with only one paper? Yikes.

        • faoiseam says:

          Different disciplines have different amounts of papers. Philosophy famously has people get professorships without a single publication.

          Examples:

          Susanna Rinard at Harvard finished her PhD in 2011, but had her first publication in 2013, Why Philosophy Can Overturn Common Sense”
          Oxford Studies in Epistemology, volume 4, 185-213 (2013).

          Adam Elga of Princeton had one publication before getting on tenure track there, “Self locating Beliefs and the Sleeping Beauty Problem”.

          Agustin Rayo of MIT had one paper, as a co-author “A puzzle about De-Rebus beliefs” when he got his MIT job.

        • tgb says:

          Depends on discipline – got my PhD in math without any publications, only submitted my first nearly a year after defending and still editing the second paper from my thesis work. It wasn’t smart, I should have gotten the first paper out much sooner since I had the result sometime in my third year and it would have made my life easier. But it’s far from unusual to graduate without anything published (though generally you need something that is close to publishable and others in math publish quite a lot). I actually switched fields for my postdoc and got my first publication in my new field before submitting my first in my phd topic.

      • johan_larson says:

        I finished my PhD in computer science with only one paper to my name. It’s possible I could have tailored the contents of my dissertation into one or two more, but since I was sick of the program by the end and headed into industry anyway, I didn’t bother. And in fairness, if doctorates had grades, mine would have been a D.

    • joshuatfox says:

      My doctoral adviser’s adviser got tenure at Harvard with 8 short papers. Even in 1990, my adviser said that it would no longer be possible to do that.

    • Nietzsche says:

      Gettier’s 91 and long retired, so maybe he shouldn’t be accused of slacking. He was a one-hit wonder, though—like Hugh Everett III who published the many-worlds interpretation of quantum mechanics in his sole scientific paper.

    • eightieshair says:

      Kenneth Wilson got tenure at Cornell without, I believe, publishing anything during his pre-tenure period (mid 1960s). Of course, when he finally did get around to publishing what he had been working on during those years, it won him the Nobel prize in physics, so not a very typical case.

    • Frog-like Sensations says:

      Here’s the paper for anyone curious. Just 3 pages!

      In fairness, it did revolutionize epistemology. But it was also anticipated by a footnote from Bertrand Russell.

      And the story we typically tell undergrads about it greatly oversells its significance. In that story, philosophers had all agreed on the JTB analysis of knowledge stretching back to Plato. This analysis was then finally disproved thousands of years later by Gettier.

      This is completely wrong even ignoring the fact that others came up with Gettier cases first. The truth is that conceptions of knowledge that are susceptible to Gettier cases only gained widespread acceptance in the decade or two prior to his article. See this paper for details on the history here.

    • Ronald Coase got a Nobel with two papers. That wasn’t everything he published, but that was what got the prize.

  2. zima says:

    The first comment on yield protection has been true of law schools for some time. I got into four or five schools that had consecutive rankings on US news and was rejected by all schools above *and* all schools below that rankings band that I’d applied to. Several of my friends had similar experiences. If colleges are doing this, it would be very concerning because college admissions is less predictable/numbers-based than law school admissions. Are there stories of smart kids getting yield-protected out of safety schools and not getting in anywhere?

    I’m not convinced there is increasing competition generally as opposed to just in specific fields. People graduating from college today have a whole new high-paying industry in tech that was much smaller ten years ago. I’m seeing people from a local engineering college going into tech startups or even non-technical roles at large tech companies and earning $150k+, whereas when I graduated from my Ivy League college in the mid-2000s, the dream was to make $75k at McKinsey. And things have certainly gotten much easier in my field of law compared to when I graduated in the Great Recession. Also, while I do have the feeling sometimes when hiring that the young people coming up the pipeline look more impressive than I was, I attribute that largely to impostor syndrome in that other people will always look more impressive on paper than you think you are.

    • Creutzer says:

      And things have certainly gotten much easier in my field of law compared to when I graduated in the Great Recession

      I would be interested to learn what your field of law is, as this is intriguingly contrary to the common perception/wisdom that law, as a field, has become overcrowded.

      • zima says:

        Corporate practice in a big law firm. Things were gruesome in the early 2010s but have recovered. Overall, there is still substantial growth over the last few decades even taking the recession into account. The standard starting salary is now about double what it was in 2000, outpacing inflation considerably. And work-life balance is generally regarded to be better today than in 2000 too.

        The overall legal profession has similar job growth and higher wage growth than the economy as a whole, and the reduction in law school applications due to the Great Recession reduces competition further. Overall new lawyers graduating today face the least competitive job market in a generation.

        • gleamingecho says:

          At least in my state, your run-of-the-mill, non-elite litigation firms (largely insurance defense) are charging nearly the same rates as they were in 1990. This doesn’t refute your point about the competitiveness of the job market, but it paints a somewhat less-rosy picture of the legal industry’s general trajectory. I wouldn’t be insulted if you wanted to argue that Big Firm Corporate Law is a different industry entirely than small-scale personal-injury litigation, however.

  3. meh says:

    Well I was just noting pew seemed to disagree with it, not sure I buy their argument. But I would believe electronic applications in general could be some of the increase. I remember applying I had to line up everything in the typewriter just so, it was somewhat of a barrier for applications… By the time I was job searching, everyone was mostly online, and I could just click apply to 10 postings in a night.

  4. meh says:

    Out of something like 20 different options surveyed, “where the candidate went to college” was rated dead last in importance.

    My intuition says there is a Bradley effect going on here.

    • j1000000 says:

      Yeah.

      I agree with those employers that “formatting of resume” is indeed more important as a signal, in that if someone writes in comic sans and the weird aLtErNaTiNg caps of late 90s AOL profiles, then I’d never hire them even if they graduated summa cum laude from Harvard. But given that 99% of applicants have acceptably formatted resumes, it’s irrelevant.

      • Lasagna says:

        You’d be surprised. I do lots of hiring, all of lawyers. At my current job I’ve probably hired 20-30 lawyers, all with 15+ years of experience from specialized, complex areas of practice. I’d also list “formatting of resume”, if broadly defined to include readability, brevity, accuracy, lack of typos, etc., as the most important factor.

        The applicants have already been filtered by recruiting before I see them, but I’ll still look at 30 resumes, minimum, per position. And if you’ve sent in a generic resume (rather than one tailored to the position), if I can’t immediately understand what your work experience is, or if, god forbid, you have a typo, it’s not likely I’ll bring you in for an interview. This really isn’t me being harsh – I can only interview a handful of people, I’ve got to select them based on something, and the only something I have is their resumes.

        And you would be amazed at how bad many of them are. Putting together a good resume takes, minimum, a dozen hours, spread out over days, per job. If you don’t take that time you’re going to screw it up, and lots of people – even high-powered, experienced attorneys – don’t do it.

        • RalMirrorAd says:

          It may be that this is a case of range restriction, all the applicants come from a similar range of schools therefore the school selection doesn’t mean much? [even though it did]

          • Lasagna says:

            That’s a good, succinct way to put it, in my case at least. At my current job – where I’m hiring highly experienced attorneys – I barely look at their law school. When I do, though, surprise surprise, they’re all from top law schools, and the “less” top the law school, the better they did there.

            Though this is just my current job. When I practiced at big NY firms, the only thing that mattered was the law school and how well you did there. Harvard, you get an offer. SUNY Albany AND on law review, you get an offer.

        • raj says:

          > if you’ve sent in a generic resume

          What exactly is meant by this? Surely you don’t expect applicants to tailor their resume to your specific company? The job application process nowadays involves hundreds of applications and dozens of interviews; it seems unreasonable to expect an individual applicant to invest significantly on the front end.

          • NoRandomWalk says:

            @raj not really.
            You’re applying to maybe 20 top firms at the most.

            If you’re midway in your career, you’re applying for roles that value different experience at several of them. You definitely want to tailor your application to highlight the experience that is more relevant.

            If I know that there are 10 interview spots, and 100 resumes, I will definitely put in a ton of work on the front end to maximize the chance I’ll be selected.

          • Lasagna says:

            That’s what I meant, yes. 🙂 The resume should be tailored to the job you’re applying to – our job posting lists the specifics of what we’re looking for; the resumes should be a response to that.

            That’s good advice in general, no matter what job you’re applying for. But also remember that the positions I’m hiring for draw from a very narrow subsection of attorneys. Most of the applicants aren’t sending out lots of resumes. They might only be applying for the one position.

          • aristides says:

            I work in HR, and I will second Lasagna’s answer. We technically accept all resumes, but we have a set format we prefer that has to answer 5 specific questions for each job. I would estimate that 10% of resumes we receive go in the trash for formatting reasons, while I completely ignore where you went to undergrad. We’ve hired many people from schools I never heard of for fairly important positions. The exception is grad school. Any JD, MD, and MBA, we look closely at that grad school.

          • Anthony says:

            My last three job searches involved three resumes, and the two successful resumes were mostly pro-forma. The third-to-last job search involved a phone call. Admittedly, it was to someone I’d worked for before, but the phone call ended with “Can you start on Monday?”

            There are advantages to being mid-career.

        • Wency says:

          I’m trying to figure out how you spend a dozen hours on a resume. Is this multiple pages, a full CV giving details about specific cases?

          I’ve had a fairly successful career in finance, and when I’ve job searched, I’d spend maybe 30-60 minutes tailoring my baseline resume to the specific job. This just means making sure your resume directly indicates that you have every skill and experience listed in the job posting. I don’t know what else I could do.

          So this might be an industry difference, or maybe I’ve just done resume-building wrong. I’ll note that I’ve never gotten a job, maybe only once or twice got past an initial screening interview with HR, without working my network (initially my school’s alumni network, later mostly former coworkers) to open up a direct channel to someone on the hiring side.

          • Lasagna says:

            Yeah, I wasn’t happy with what I wrote there. It was way too glib. What I should have written was “if you’re starting from scratch…”, but obviously most people aren’t. It hurt my argument.

            I think my basic point stands, though. You should (1) tailor your resume for the job that you’re applying to, (2) that takes a lot of time, and (3) it should be done over the course of a few days. Work on it, put it down, come back the next day, revise, do it again.

            But like you said I’m coming from a very specific industry here. I’ve never hired anyone in any other context, and it’s been, Jesus, twenty-one years since I’ve applied for a non-legal job. So it might be different elsewhere.

            For what it’s worth, in the hiring I’ve done and the jobs I’ve gotten: contacts and networking are never a bad thing, but they aren’t necessary. I’ve had three jobs since law school and I got them all through applying cold (though I went through a legal recruiter for the last two). And we hire people all the time with no internal contacts where I work now.

          • A Definite Beta Guy says:

            Meh, takes me about 10-15 minutes to make a new resume. I have practically every conceivable bullet point for every position I was in, for any position I might reasonably want to apply to.
            After that, it’s just copy-paste the relevant bullet points into the resume and make some small adjustments.

            On the up-front side, it probably took around 12 hours to come UP with all the bullet points and format appropriately. But with the up-front work complete, it’s just plug-and-play.

          • Cliff says:

            Putting together a good resume takes, minimum, a dozen hours, spread out over days, per job.

            What is your basis for thinking this? Have you done it? If the resume has the relevant information about their education and work experience, that’s sufficient. I see no reason why that should take 12 hours to produce.

            I also hire attorneys for specialized positions and it’s fairly apparent to me from the resume whether they have the skills that are needed or not. Just how good they are is literally impossible to tell without looking at a lot of work product or working with them. Nothing on a resume can tell you that.

            Resume formatting is meaningless.

      • Urstoff says:

        It is very much not the case that 99% of applicants have acceptably formatted resumes; extend that to cover letters, and you’ve already got a strong first filter to use for hiring.

    • eqdw says:

      I agree with that. The survey said what it said, but the results were very surprising.

      I would believe it if ‘where they went to college’ was near the bottom. But as dead last, it’s suspicious.

      One interpretation that confirms the data but also our pre-existing notions: perhaps “where you went to college” is not actually the thing employers are selecting on, but they are selecting on something that correlates heavily with that. Speaking from past experience, I used to work at a company where fully 1/3rd of the staff were Dartmouth graduates. There was absolutely no hiring bias in favour of people who went to Dartmouth, but what there was was a bias for personal referrals. The founder went to Dartmouth, so the first five or ten people he hired were colleagues from Dartmouth. Then they had a company with ten Dartmouth grads and when they wanted to hire they did personal referrals. Next thing you know, you have 100 people at your company, and 40 of them are Dartmouth grads, not because you were preferentially hiring Dartmouth grads, but because all your existing staff just referred people they knew.

      If something like this is driving things, then it would be actually true that employers don’t care about where you went to college, but at the same time it would also be true that there is heavy over-representation of people who went to elite colleges at various companies.

      • Anthony says:

        There are a *lot* of companies hiring a *lot* of employees. Many of those companies are getting zero resumes from Harvard, or even HYSP grads. For a whole lot of jobs, the mere fact you completed college shows that you are literate enough and conformist enough to do the job. Not every job needs the absolute brightest and most diligent.

        Also, when hiring people with an evaluatable work record, things in the distant past like their college are much less relevant.

        On the other hand, at my last job, there were too many Ph.D.s, and a lot of intellectual snobbery – we didn’t care as much about the university someone went to, as who their adviser was.

  5. Jack V says:

    I’ve been introspecting about something similar and haven’t come to any firm conclusions.

    My best guess is something like, society has in some ways become less stratified with more social mobility, but not as far as it likes to pretend. So maybe 100 years ago, most people got a similar sort of job to their parents, and if things were bad, you maybe kind of just starved. And most people didn’t think of “become an academic” or “become a lawyer” as something that was likely for them, even if they would have had the capacity. And if you were ambitious, you might work to earn more money and save up and educate yourself as you went along, rather than feel like everything depended on your exams at 18.

    But now, everyone feels like everyone is sorted by ability at age 18, so everyone tries for the best degree/job/house they could plausibly get, but that means there’s constant competition, because would try.

    I’ve noticed it particularly in academic jobs because science funding dropped a lot, so a lot of the best-of-the-best people who might previously have had quite prestigious jobs at top universities, can’t get any job related to their phd at all. And it might be fairer to have a lottery and say “ok, we’ll accept 1/3 the number of people, but then we’ll fund their research properly”, but no-one wants to do that, so instead, as many people as possible are accepted into some sort of academic job based on extreme competition, and then they’re slowly weeded out and told its their own fault, when there’s not enough funding to go round 🙁

    But I suspect even jobs in fields that haven’t shrunk feel similarly

  6. Douglas Knight says:

    Byron White:
    Wikipedia has a citation. The citation doesn’t mention Harvard or illness, but it does say that he was admitted to Yale a week after classes started, which is the relevant detail. On the other hand, it says that he was kicked out of evacuated from Oxford because of the war, which is a pretty good excuse for applying at the last minute.

    • Steve Sailer says:

      When Byron “Whizzer” White started law school in 1939, he was a superstar football player, runner-up for the 1937 Heisman Trophy and leading ground gainer in the NFL in 1938. Celebrity has its privileges. It’s kind of like if Patrick Mahomes had a high LSAT score and wanted to start law school in the spring semester after the NFL playoffs: strings would be pulled to accommodate him.

  7. rlms says:

    If Steve Sailer is right that places for undergraduates at elite colleges haven’t increased in line with population growth, I think that explains most of it. If getting into Harvard today requires being in the top 70% of 1986’s successful applicants, that seems like a pretty significant difference.

    • mobile says:

      There are also more quality international applicants than there were in the 80s.

      • Steve Sailer says:

        When I applied to college in 1976, the world’s median East Asian 17 year old was slopping pigs on a collective farm in Red China. Today they are checking out USNWR rankings for American colleges on a smartphone.

  8. Deiseach says:

    In a world where there are 10 places at Harvard every year but 9 applicants, everyone who can read Latin gets in, and no-one bothers to study harder than what it takes them to meet the minimum requirement.

    I think there is also a minor but important point, arising out of the “Compare Canada with teeny population and lots of college places versus the US with high population and restricted places” – the growth in population.

    Back in 1900, the population of the US was roughly 76 million. Harvard etc. were only expecting “top ten Latin speakers” and built everything (from physical structure to culture around how classes were taught etc.) around that presumption. In 1900, maybe you could reasonably expect 9 people to be the top Latin or whatever in a population of 76 million.

    But with a population of 315 million, by sheer numbers, there are going to be more than 9 or 10 or even 11 top Whatever students. You’re still physically and culturally set up to cope with “top ten” but now you have Top Twenty applying – of course you’re going to pick and choose! Especially as picking and choosing allows you to keep the reputation of being the elite, the Only Really Smart People Get In Here that makes your degrees so valuable (whether you consider that as educational value or only as signalling).

    And I think that trickles down to all the rungs of the third level ladder. I imagine it’d be possible to build a brand spanking new university that could accommodate thirty thousand undergrads, but who’s going to do it? You’d need a hefty endowment to set it up, which means either a philanthropic multi-billionaire willing to throw money at it for at least a decade, or the government (and I’ve seen enough online pissing and moaning already of the “could this ointment not have been sold for the poor?” around billionaires offering funding to restore Notre Dame that I imagine the same would happen about opening a new mega-university, so any billionaires out there are going to spend their cash on Approved Projects instead; ditto the government taking on such a huge capital project).

    Nevermind that your new mega-university has no reputation, so people will still hedge their bets by the Best and Brightest continuing to apply to the Ivies so it will take a long time to build up the reputation and background of “no honest, Mega U is every bit as high status as Yale”.

    The University of Toronto example quoted is a bit misleading as well; looking it up (to see who endowed/funded it), I see it’s yet another Started Off As A Seminary effort, and the history seems to be that it got to its current figure of nearly 70 thousand undergrads served by absorbing a lot of independent colleges as well as founding various schools under its own aegis. Think of it as if Harvard had taken over a lot of separate colleges under its wing and absorbed them into the Big University brand, instead of having all the small private liberal arts colleges flying their own way.

    So it’s certainly not like University of Toronto started out planned as Mega-University going to handle 70, 000 undergrads; I imagine for accreditation etc. the ‘brand’ of the University was considered valuable enough for Private College to give up issuing its own degrees and be willing to be absorbed as “constituent college of the University”:

    Unlike most North American institutions, the University of Toronto is a collegiate university with a model that resembles those of the University of Cambridge and the University of Oxford in Britain. The colleges hold substantial autonomy over admissions, scholarships, programs and other academic and financial affairs, in addition to the housing and social duties of typical residential colleges.

    Under this model, Harvard could certainly grow to encompass 30,000 undergrads if it built separate new colleges and came to agreements with Small Liberal Arts College in Wisconsin and other states that now they would be Constituent College of Harvard University, but everyone would still think of “Harvard” as Actual Harvard and not “You went to Wisconsin Harvard”.

    This is the model that the National University of Ireland operated under; three separate colleges in different cities (Cork, Galway and Dublin) all University Colleges and forming the National University of Ireland (Trinity is its own separate case).

    • eric23 says:

      1) This sounds like the state university model. And yet nobody considers UC-Berkeley and UC-Merced to be interchangeable.
      2) There is already a simple “brand” for nearly all the top US universities – the Ivy League. Harvard is about 50x smaller than McGill (relative to population size), but the Ivies as a group are only 5x smaller than McGill – much more comparable.

    • I-go-here says:

      This isn’t an accurate account of how U of T got so big. The colleges do not control admissions to the university. The university is large because it’s grown 50% in the last twenty years and is planning to keep up that rate. Nobody lives in residence, so it’s not talent constrained. The university has a good reputation in East Asia, so the growth is sustainable by admitting students who pay very large international fees.

  9. AnthonyC says:

    I don’t know what rhetoric they use now, but when I applied in 2004 (for Class of ’09) the Harvard admissions department was pretty open (in person at least) about the fact that they could easily fill their class slots four times over without sacrificing any metric of quality that they could name or specify. They’re not making it a secret that selectivity at that level isn’t meaningfully correlated with ability.

    • Eponymous says:

      I thought the data from the recent lawsuit about their admission process contradicted this claim, at least if you go by their own measure of academic ability.

      • Anthony says:

        I think what the other Anthony is saying is that even with discrimination against Asians, Hahvahd could quadruple the size of its freshman class without sacrificing what it considers quality.

        Incidentally, the reason Hahvahd discriminates against Asians and really does look beyond academic ability, is that they are self-consciously preparing America’s future elite. And America’s elite won’t be very Asian for the forseeable future, and will never be full of bookish nerds. California’s may be, but preparing California’s elite is Stanford’s job.

    • the Harvard admissions department was pretty open (in person at least) about the fact that they could easily fill their class slots four times over without sacrificing any metric of quality that they could name or specify.

      I don’t believe it.

      When I went to Harvard, back in the early sixties, there was an obvious sizable range of ability among the students, some of which could have been picked up by metrics available at admission. Are they claiming that everyone they admitted and three times as many that they didn’t had double 800’s on their SATs? Best possible scores on all their AP courses?

      I think it’s still the case that some physics majors take the physics 11/55 sequence, and some take the alternative less elite sequence, and similarly for math. Somehow they are distinguishing.

      • anonymousskimmer says:

        Are they claiming that everyone they admitted and three times as many that they didn’t had double 800’s on their SATs? Best possible scores on all their AP courses?

        You’re forgetting (or don’t know of) ceiling bumping.

        http://megasociety.org/noesis/167/4.htm

        a scarcity of near-ceiling scores is often due to “ceiling bumping,” composed of errors in the test itself and careless errors on the part of the testee, which artificially limits the performance of the highly capable.

        • The implication of that is that the person with 800 and 790 might be as good as the person with 800 and 800. That’s just the uncertainty one always has to deal with–the person with 700 and 700 might be as good too, having taken the exam after a week of sleepless nights due to breaking up with his girlfriend.

          On average, the 800/800 is better than the other two, so hiring the latter instead of the former is, probabilistically speaking, sacrificing some academic quality.

          • anonymousskimmer says:

            Responding to your original critique: Realistically, how many students pursue majors which benefit from higher skill in both English and Math? If you don’t need the skill in superabundance for your major (only as a threshold, beyond which no academically discernible boost exists), then “without sacrificing any metric of quality that they could name or specify” is technically true.

            From College Confidential:

            Last time I checked (I think these were the stats for college-bound seniors in 2009):
            Slightly under 5000 people got 800’s on Writing
            About 10,000 got 800’s on Math
            About 8,000 got 800’s on reading

            You mention “hiring”, they’re talking about academics. And as everyone should know, the SAT isn’t an IQ test (which break out many more subskills), and math and english skills aren’t the only important cognitive abilities in real jobs.

          • faoiseam says:

            Not that many people have perfect SATs. In 2013 494 people got 2400. 1576 got above 2350.

            People greatly overestimate how many people have perfect SATs. College Confidential is the worst at that. Reading there, you would think that perfect SATs, multiple papers in Science, and IMO medals were given out like candy.

          • Responding to your original critique: Realistically, how many students pursue majors which benefit from higher skill in both English and Math?

            I would guess that, for most majors and almost all careers, skills in both English and Math are of some value, although obviously not equal value.

            But in any case, Harvard doesn’t know which applicants will end up in what major, which makes your point irrelevant with regard to admission.

      • Steve Sailer says:

        Steven Pinker wrote an article for TNR a number of years ago proposing that Harvard focus solely on academic talent.

        This would probably take a new test with a much higher ceiling. Back before 1995, the SAT-Verbal was very hard. White SAT-Math 800s were not all that rare, SAT-Verbals were. In a typical year, only about 9 kids nationally would score a perfect two 800s.

        At present, Harvard seems content with metrics that don’t distinguish the future Pinker from the future Buttiegeg, allowing them to choose the top tier student more likely to donate a lot of money or become a celebrity.

        • johan_larson says:

          Are there any colleges that admit students purely based on academics? Or even just academics and ability to pay tuition? Caltech is supposed to be very academically focused, but I don’t know whether their admission decisions are only based on academic criteria.

          • faoiseam says:

            A admission committee member writes on quora:

            We are looking for people who have demonstrated a drive to be a scientist, for people who have gone well above and beyond their schoolwork and have engaged in a program of teaching themselves, because by they have always known more than their teachers. We look for people who have created something: maybe you program computer games, you build physics experiments in your garage, you discovered a comet, you derived calculus on your own, you have a bacteria collection, or you’ve read Galileo in the original Italian. Whatever you’ve done to make yourself stick out as an amazing potential scientist and philosopher, we want to hear about it. We want to hear the story of you, why you need to excel, and why you have to sponge up every concept that comes your way.

            If we eliminate all the folders of the applicants who are merely good students and had some extracurricular activities, and keep only the ones I’ve talked about, the exciting ones, we still are left with about 750 folders for 200 spots. So we have our pick of who we want to join our community. Of those applicants, we select the ones we think will fit that community best. Read the Caltech web pages— both the official ones and the ones the students put up. Explain how you would fit in with that community. What does the Caltech Honor System mean to you? What would you contribute to the Interhouse party? What Hovse would you live in? Why? We want people who will interact with each other and explore the world together. You’ve told us why you’re a special student and scientist, now tell us why you’re a special person too. Again, well-rounded individuals need not apply. We’re looking for special people, spiky people.

            Essentially, they admit people who have done well at school, and are willing to tell lies about their interests in math and science. If you are willing to make up a story about how you independently developed calculus, or taught yourself a foreign language to read Galileo (famously, Joyce taught himself Norwegian to write to Ibsen, which is the source of this story, I would guess.), then Caltech is for you.

            Also, stories about who you are are important to them. This is a huge tell to asking people to self identify so that people who look like us can be admitted. In this case “us” is the admission department, not the school.

            They also ask for high schoolers who have discovered comets. A cursory search of the web seems to indicate there are not such people. You might as well ask for high schoolers who have discovered a new continent.

      • j r says:

        I don’t believe it.

        When I went to Harvard, back in the early sixties, there was an obvious sizable range of ability among the students, some of which could have been picked up by metrics available at admission.

        What percentage of the class were legacies, athletes, or the children of the wealthy and famous? That would probably account for the spread right there.

        There are more than enough students that would succeed, even excel, at Harvard, if Harvard decided to quadruple the size of its next incoming class. These effects wouldn’t be felt at the top tier schools, but probably at the margins of the second and third-tier schools.

        • I doubt that a majority of the students were legacies etc., but a sizable majority, as best I could tell, were noticeably less able than the best students.

          As I mentioned earlier, in a number of fields there was a two year sequence (11/55) for the better students and a three year sequence for the majority.

          There are more than enough students that would succeed, even excel, at Harvard, if Harvard decided to quadruple the size of its next incoming class.

          How do you know?

          I can’t report on the current state of Harvard, but I spent four years there in the early sixties and am confident that was not close to true then.

          • j r says:

            How do you know?

            I can’t report on the current state of Harvard, but I spent four years there in the early sixties and am confident that was not close to true then.

            My working assumption is that if Harvard quadrupled its entering class, it would be taking all of those extra students from the pool that would otherwise be going to YPS or one of the other tier-one schools?

            Do you really believe that the median student at Yale or Stanford and the best students at Cornell or UPenn or U of Chicago, etc. would be failures at Harvard?

          • Do you really believe that the median student at Yale or Stanford and the best students at Cornell or UPenn or U of Chicago, etc. would be failures at Harvard?

            The phrase was “succeed, even excel”

            The best students at Chicago would be comparable to the best students at Harvard—but might prefer Chicago. The median student at Yale would be comparable to the median student at Harvard, and might prefer Yale.

            But the median student at Harvard doesn’t excel.

            I agree that if the students at Harvard’s peer universities all preferred to go to Harvard instead, Harvard could greatly increase admissions while maintaining about the current distribution of abilities—was that your point? But under those circumstances, Harvard could raise its admission standards and eliminate the bottom half of its present class. So there is still a tradeoff.

          • j r says:

            was that your point?

            No. My point was that you could quadruple the size of Harvard’s next entering class and there would be almost no tradeoff. There are enough able students in the overall candidate pool that you could expand that pool several times without noticing a difference in academic performance.

            If you quadrupled the next entering class of HYPS, again, I think that there would be almost no tradeoff. Those schools would simply absorb all of the student that would have otherwise gone to other top tier institutions and those institutions would, in turn, absorb the kids just on the other side of the cutoff.

            My point is, at that level, the number of spots relative to the number of qualified applicants is so constrained that a lot of kids are probably losing out on the basis of what amounts to a coin flip (e.g. candidate A has everything that candidate B has, but candidate A is from California and candidate B is from Alaska, so we’ll take B because we already have too many kids from California).

            If you quadrupled the next entering class of the top 20 undergrad programs in the U.S., you still probably wouldn’t see any tradeoffs at those schools but you would might start to see effects on the margins of the next 20 schools on the list. That is, second tier schools would lose all the kids that would have been their best performers and their student bodies would be made up of a lot more “average” performers.

          • My point is, at that level, the number of spots relative to the number of qualified applicants is so constrained that a lot of kids are probably losing out on the basis of what amounts to a coin flip

            Probably true, but, if the admissions people are competent, those are applicants who would be in the bottom half of the class. So letting them in instead of the more qualified applicants, or expanding admissions to let them in in addition, would lower the average quality of the student body.

            Do you disagree? Are you claiming either that that admissions people have no ability to evaluate academic ability at relatively high levels or that there are so many students at the very top level that Harvard could limit itself to them and still accept many more students than it does?

            Both of those strike me as implausible.

          • j r says:

            Do you disagree? Are you claiming either that that admissions people have no ability to evaluate academic ability at relatively high levels or that there are so many students at the very top level that Harvard could limit itself to them and still accept many more students than it does?

            Both of those strike me as implausible.

            Yes, I disagree. I believe that there are more than 2000 graduating high school seniors in the world (or even just in the US) who could do well academically at Harvard.

            There is nothing implausible about that. Harvard’s selectivity is a feature not a bug. And it is especially not implausible when you take into account population growth and the increase in students on a college prep track.

          • I believe that there are more than 2000 graduating high school seniors in the world (or even just in the US) who could do well academically at Harvard.

            1. Who would choose to go to Harvard? Lots of those students are currently going to Stanford or Yale or Chicago or MIT or Cal Tech, each of which will be viewed by some students as better for them than Harvard.

            2. Who would do as well academically as the best tenth of those currently attending Harvard? The best half?

          • Clutzy says:

            I think a large part of Harvard (and other institutions) selectivity in the bottom half of the class isn’t particularly related to the ability to excel academically. They are there for other reasons.

            I know 2 people who got into Stanford from my HS, the first was hispanic who wrestled on the team, the other was half black- half American Indian. Neither did well academically there (5.5 years to graduate) and both are high school teachers now. Neither would have been in the top 15 of my selections for their graduating class. Two much better guys I knew got waitlisted. One went to Notre Dame and works at Bain, the other went to Northwestern, has already managed a successful congressional campaign and works at a big K-street firm. Even I would have been a better selection (although I didn’t even apply to Stanford) with my decent legal career in patent law following some bioengineering work.

            So, maybe the departments are idiots, or maybe they have other goals than having a full class of smarties, because they already have enough real smart people in their top 10% that the endowment fund is never going to dry up.

    • faoiseam says:

      the Harvard admissions department was pretty open (in person at least) about the fact that they could easily fill their class slots four times over without sacrificing any metric of quality that they could name or specify.

      That is why the admission department should be abolished. Every professor bemoans the quality of the incoming class. Half the students should not have been admitted. The admissions department is just unable to tell who actually is smart.

      • johan_larson says:

        I can easily believe some colleges have that problem. But it’s hard to believe the elite colleges have it too.

        • But it’s hard to believe the elite colleges have it too.

          I have been an undergraduate at one elite college (Harvard), a grad student at another (Chicago), and taught at at least three (Chicago, Stanford, UCLA).

          In all of those cases, there was a considerable range of ability among the students. Unless things have changed sharply over the last twenty years or so, I expect there still is.

          • faoiseam says:

            I have not been at all those colleges, but at the ones I am familiar with there is a clear gradient in ability that continues to the very top of the class. It is not uncommon for a student to show up who is widely considered to be the best student that has been seen in a few years (Obviously, this only happens every few years). The professors generally agree about which student is the best, and this is visible almost immediately.

            Once you get into the middle of the student group, distinctions are hard to draw, but that the top of the class, there generally is a fairly clear ordering in terms of raw potential.

            Students own estimate of who is smart generally agree with the faculty. The only people who cannot tell who is smart are admission officers and high school teachers, as far as I can tell.

          • johan_larson says:

            We agree there are real differences in intellectual ability among students even at elite institutions. That’s not the issue.

            What I’m responding to is this bit:

            Every professor bemoans the quality of the incoming class. Half the students should not have been admitted.

            Could this really be going on? At an elite institution? I would expect that given how carefully candidates are screened, and how high the standards for admission are, even a fifth percentile student at a place like Stanford would still be quite clever and diligent.

          • rlms says:

            Even if students were selected entirely based on academic ability, I’d expect >1% to be able to appear significantly smarter on paper than they actually are (either through being unusually good at standardised tests, cheating, or something else), and a higher proportion to discover in college that partying is more fun than coursework (so even if they have the ability it isn’t displayed). But at American colleges I’d think legacies, affirmative action and athletic scholarships would be more important.

          • faoiseam says:

            I think the issue is that half the students are in the bottom half of the class, and very clearly in the bottom half. It is not a case of there being a middle group of students who are generally competent, and a few great students, and a few legacy cases. If there were only a few students out of the mainstream, then there would not really be a problem. Instead of this, a large portion of the class is taken from students who are not as academically ready for college as the others.

            I presume this is because Harvard will happily take a child with 600s on their SATs if the child is a recruited athlete (or so my child was informed by Harvard athletic department), and will take donors kids at that level or below. Hispanic kids are admitted with strange pedigrees. I have helped almost functionally illiterate children with their application essays that got them into USC. Black students are very clearly divided into two groups, the Africans, who are generally middle to bottom quartile, and the non-Africans, who are wise enough to not take rigorous classes.

            International students also have the possibility of being absolutely terrible. A college whose board affairs I am familiar with had the issue that they admitted a substantial number of Chinese international students who scored well on tests, only to find they required extensive tutoring to stay in the program. Colleges really care about dropouts, so a large amount of money was spent to handhold these international students who were not capable of the material. (You would think it would be easier to just get the professors to pass them, but professors are really unreasonable about things like that.)

          • Could this really be going on? At an elite institution? I would expect that given how carefully candidates are screened, and how high the standards for admission are, even a fifth percentile student at a place like Stanford would still be quite clever and diligent.

            The problem is that “should not have been admitted” isn’t an objective criterion. From standpoint of the professor, the class would be much better if everyone was as good as the top half of the students, so the bottom half shouldn’t have been admitted.

          • arch1 says:

            FWIW Caltech has long been (and I think still is) an interesting anomaly in that its 25th %ile SAT score is about 100-ish points higher than the next highest institution. I don’t know the full story but I’m guessing this is because they have a high effective cutoff on the math side, as the curriculum tends to be quite hard-sciences focused thus about as math-intensive as any undergrad institution anywhere.
            When I was there, there were still quite a few folks who struggled in math, alas.

      • Steve Sailer says:

        As Richard Armour wrote, the Admissions Department is in charge of admitting the college’s mistakes.

      • Steve Sailer says:

        Harvard’s Admissions Department obviously made a mistake in not admitting Scott, who is eventually going to become the kind of famous intellectual of whom Harvard would like to have a monopoly.

    • Cliff says:

      I’m sure they say it, but it’s obviously false. Just look at the quality metrics for the students, they are far from perfect. What they are trying to elide is that they’re not trying to admit the most qualified but the most likely to donate large sums of money or to end up in positions of power, while achieving their publicity/diversity goals.

  10. Eponymous says:

    I’d heard this before; my impression is that a big part of consulting is having prestigious-looking people tell you what you want to hear. If what they’re actually hiring for is prestige rather than competence per se, that could make it a special case

    This impression is completely wrong about the kind of consulting I did after undergrad (econ), which was extremely competence-based. They hired almost exclusively out of top schools because they wanted top quality, since errors in this line of work could be so costly.

    I’ve now attended and taught at universities ranging from the very top to big state U types. There’s a precipitous dropoff in student quality, although the top students at big state U are still very good. This probably doesn’t matter much if you’re hiring people 10 years out from college with a work history. It matters a lot when you’re hiring right out of college. And careers are path-dependent.

    One problem is that if you attend a mid-level college there’s no way to credibly signal that you’re actually a top-level talent due to grade inflation and curves. It’s like if the top score on the SAT was 1400.

    • DragonMilk says:

      Agreed – a lot of comments seem to focus from the over-achieving high school student perspective.

      From an employer’s perspective, where someone went to college is an easy step 1 filter if you have tons of applicants already. Just as colleges will use standardized test scores as a filter before bothering to read essays, companies choose to filter colleges before granting interviews. Both have decided that there are enough qualified candidates in the remaining pool vs. wasting time digging through a mound of nopes to find someone worthwhile. I don’t see any issue with that.

      • gbdub says:

        Our company doesn’t filter resumes – but we only do in-person recruiting at a relatively small number of schools (nationally elite and schools close to our major sites) and the vast majority of our fresh out of college hires come from that program.

    • A Definite Beta Guy says:

      Yeah, from my perspective:

      1. Where you went to school definitely mattered getting a position out of school. A lot of high earning jobs were only available to people from certain schools, particularly at consulting places or PE firms or other prestigious firms. You definitely couldn’t get from my school into, say, Deloitte consulting. Meanwhile, my friend’s girlfriend majored in something like English and walked into Accenture business consulting, probably in no small part because she went to Northwestern. And that was her safety option.
      2. This absolutely makes sense because there is a huge difference in student quality between the schools. I went to a pretty damn good IL public high school, which are basically private schools with the amount of money they spend. I was a decent student in my trailing years, but the few kids who went the Ivies were head and shoulders above me. Not just intelligence, but actual work ethic.
      3. Big State U at least gave you an opportunity at some of the top positions. UW-Madison, Ann Arbor, and Illinois-Urbana are all highly respected schools. They gave you a foot in most doors, but you had to prove yourself.
      4. There are still plenty of financially viable pathways for students graduating from good state and below universities. I went to a second tier state school and several of my friends are clearing six figures now. It takes some hard work and some good career moves. My salary between 2016 and 2020 will probably double due to the same combination, though I would’ve been unemployed and fucked had I stayed in my 2016 job. Sometimes you just need to roll the dice.
      4b. Understandably those salaries are probably not all that impressive to some of the crew here, but that’s legitimately good money for most of America.
      5. There are a lot of people at second-tier state schools who should not be there and are currently in shitty jobs. They were, however, mediocre or shitty students.
      6. Graduate degrees like MBAs mean jack-shit unless they come from good schools and you are already advancing in your career. I know plenty of people who were stuck in dead-end jobs and got MBAs from no-name schools, and have gone practically nowhere. One quit his accounting job to become an electrician, because it’s quite frankly a better career path for him.
      EDIT:
      7. There are a lot of smart kids who went to good schools and haven’t accomplished all that much. Too much of this Disney “follow your passion” crap. I guess if that’s your dream, go for it, but you’re leaving actual cold, hard cash on the table.
      8. There are a lot of smart kids who have passed up viable career options to become staff at tech. The people on this forum are smart and hardworking enough to survive Big 4 Audit experience. These people are frequently placed into Fortune 500 companies to audit them, and then are frequently poached to become senior leadership at said company.
      If I have kids brainy enough to get into Harvard or whatever, that’s what I’ll advise, because that’s an easy way to become Senior VP or CFO by the time they are 40, and then you can easily retire at age 60 with enough money in the bank to Scrooge McDuck every morning.

      • DragonMilk says:

        Do you have kids yet?

        After reading Scott’s posts on the Hungarian chess genius raising guy, I want to homeschool (future) kids, but my friends say that they do not doubt that they will be smart, but lacking in social skills, haha

        • A Definite Beta Guy says:

          Not yet, but hopefully soon. Our kids will probably be decently smart, but I highly doubt that they are going to be Harvard-smart. My wife’s family and my family are mostly Big State U smart, and I am a small outlier in my family.

          I can’t imagine homeschooling offering much help if the kids were going to go for Ivy League, compared to going to the local high schools here. Honestly, my hope is that they don’t run themselves ragged doing 10 million extracurriculars and they have a greater level of maturity than their aunts and uncles. Oh, and that they don’t accumulate a massive amount of college debt. The ideal amount of college debt is 0.

          • The two kids of my present marriage were home unschooled. One went to Oberlin, then transferred to Chicago. The other went to Chicago.

            That looks like evidence for homeschooled kids getting into elite schools, but it may not be very good evidence. My wife, her sister, and both her parents went to Oberlin. I, both my parents, and my uncle, have multiple Chicago connections. So both of those might count as legacy admissions. The one school one of our kids was admitted to without any special advantages was Saint Olaf, which is a half step down from Oberlin in reputation, although perhaps not in quality.

            They not only accepted one of ours, they offered her money. My conjecture is that they want to replace Oberlin as the elite liberal arts college combined with a professional level music program, and it occurred to them that home schooled applicants provided a source of top students that other schools were missing.

            On the social question, my kids could probably comment better than I could. They were acculturated to our family culture, hence to some degree foreigners in college culture. But I think that was equally true of me as well, and I wasn’t homeschooled.

            The one product of home schooling I know who is close to my age feels a little socially clumsy to me–but he ended up running the Federalist Society, so whatever his social limitations, they were not crippling.

    • Cliff says:

      due to grade inflation and curves

      I have heard this is actually worse at the elite schools.

    • Steve Sailer says:

      I got my MBA at UCLA in 1980-82. UCLA had been ranked #8 in the country in a couple of surveys of B-Schools in the 1970s, but it was likely falling below that level because it’s hard to justify at a public university the really lavish funding that private B-Schools engage in. I was called in by McKinsey, the top consulting company, for an interview with the head partner of the L.A. office. He said they didn’t have open interviewing at UCLA, but they thought my resume and one other student’s looked promising.

      But, I blew the interview.

      • So we overlapped at UCLA. Did you take any courses in the econ department?

        • Steve Sailer says:

          I can recall reading an article about you in “The Daily Bruin.”

          I had a summer job in 1981 working for UCLA vice chancellor C.Z. Wilson, who I think was in econ before he went into administration.

  11. JPNunez says:

    I wouldn’t lend too much weight to employers saying themselves in what order they consider different factors to hire someone.

    • Cliff says:

      Why?

      • anonymousskimmer says:

        The cognitive error of being blind to your own implicit biases.

        You’d want a sociologist who studies employers to be telling you what factors employers use in hiring, and in what order they preference those factors (as well as which factors are statistically irrelevant).

        • AG says:

          Yep. Unless they can show you the spreadsheet calculator with the weighting values that they don’t get to fudge with at all, there’s no way to verify how much whatever factors they consider impact their evaluation of a candidate.

        • Cliff says:

          Please tell me this is sarcasm

  12. deluks917 says:

    The standard for jobs in Effective Altruism seems to have risen incredibly rapidly in the last few years. https://forum.effectivealtruism.org/posts/jmbP9rwXncfa32seH/after-one-year-of-applying-for-ea-jobs-it-is-really-really. Things can get much more competitive rapidly even in healthy systems. Talented people want prestige and talented coworkers. Standards can spiral up rapidly.

    • Erusian says:

      Reading this, my thought is that EA is culturally sexy and it’s probably something of a passion/external benefit market. Basically a market with hugely non-job benefits (like social prestige, warm fuzzy feelings, whatever.) Certain markets are so full of people who just want to work there that they get to be extremely picky. It’s also nearly impossible to get work at craft beer shops, certain sorts of religious organizations, almost anything having to do with acting, many forms of modeling.

  13. gbdub says:

    Most of the people in media writing about this phenomenon live somewhere in New England, where there simply aren’t the high quality, nationally recognizable Big State Unis that they have elsewhere in the country (particularly the Midwest, California, and the southern part of the Atlantic seaboard). In USNews’ top 25 public universities, Rutgers (17) and UConn (22) are the only New England schools (UMAss Amherst just misses at 26).

    The increasing competitiveness is clearly real, but the panic over it might be driven by people who really do find themselves in the position of “Ivy League or bust”, because your “safety schools” are a real step down and your other options (good out of state publics and smaller privates) are just as expensive and nearly as competitive for out-of-staters.

    Meanwhile, at my small high school in Michigan in the early 2000s, the top students all applied to a few elite schools (Ivies, MIT, maybe Stanford) and Michigan, confident they’d get into Michigan at least (and that’s where most of us ended up). Your “safety school” might be Michigan State, which is generally considered down a tier from UofM but still is as good or better than most New England state schools (and certainly has a bigger alumni network). (Then again, even Michigan has gotten twice as selective since I applied)

    Likewise if you’re a top kid in Chicago – U of Illinois is a damn good fallback if you can’t get into the Ivies, NW, or UofChicago (or can’t/won’t pay their premium price)

    Even in California, it sounds like top students, non-Tiger Family edition might have to settle all the way down for Irvine instead of Berkeley, UCLA, or Stanford – the horror!

    It would be interesting to compare the application habits of the top 10% of high school students in various regions of the country. I suspect there could be some significant and telling variations.

    • AG says:

      But then doesn’t this mean there’s money/talent being left on the table for someone to Moneyball, whether that’s a university or a company?

      • JPNunez says:

        Sure?

        But a company needs to identify the otherwise-Ivy-League students that went to a safety school. Dunno if colleges can look at what other colleges someone applied to? Sounds doubtful? But if they could maybe assigning a small extra if they applied to better colleges.

        The first one is doable, but normally you will need to spend money hiring a person for a while to see if he performs well anyway.

      • gbdub says:

        I’m not sure what you mean… the “safety schools” are already getting the “Moneyball” candidates. Other than from the very snobbiest of firms, the top students at high-end publics are getting plenty of job offers.

        Is there a niche for an affordable, slightly less selective option for east coasters who get rejected from the Ivies, are too good for SUNY, but can’t afford out of state tuition at Michigan? Maybe, but how do you fill that? You can’t create institutional prestige from scratch, and if it was easy, the SUNY system probably would have done it already.

        • AG says:

          So, your comment supposes that several East coast states have plenty of respectable students who are stuck between Ivies they can’t get into and expensive out-of-state universities.

          Seems like there’s a great opportunity for someone to either found a university on the East coast to scoop these students up, or for a company to hire them and pay them less than they would someone with a degree. (Of a combination of both, some sort of work-study program)

        • johan_larson says:

          SUNY-Stony Brook is #80 on the USNWR ranking of National Universities. If that’s not good enough, and we exclude #1-20 as too competitive, what does that leave us that’s on the cheaper side? (Assuming out-of-state tuition.)

          UNC-Chapel Hill is #30 with tuition at $35K.
          Georgia Tech is #35 with tuition at $33K.
          University of Florida is also #35 with tuition at $29K.
          University of Georgia is #46 with tuition at $30K.
          University of Illinois Urbana-Champain is also #46 with tuition at $32K.

          Looks like the best options are out-of-state flagships. They’re still cheaper than private colleges of roughly the same rank.

          And of course these tuition costs are before grants and scholarships and whatnot. I understand lots of students pay less.

  14. JohnBuridan says:

    What causes 60% of the 70% of students who go to college to never finish in a timely manner? This.

    My local neighborhood around 2008 started to wobble towards a lower demographic and property values started decreased. The local public school administration saw this problem and responded logically. A few administrators and teachers systematically changed everyone’s grades upwards, vouched for the excellence of the students, and sent them off to college. Almost no one in that cohort finished college, BUT the property values in the neighborhood went up (since school performance and property values are basically equivalent here).

    Besides being a classy example of manipulated causation, (lie, screw over nearly a thousand students, in order to increase property values and “student quality”), it’s also an example of how screwed up incentive structures get gamed by desperate administrators upon whom the municipality depends for tax revenue.

    • Randy M says:

      What’s that saying? Once a goal becomes quantifiable, it is no longer a good goal?

    • Douglas Knight says:

      That’s a great story of what’s possible and where we might be heading.

      But it’s great because it’s different from what I’ve heard. I’ll keep in mind to see if it’s typical, but for the moment I doubt it’s typical, and thus I’m skeptical that it answers your framing question.

    • RicardoCruz says:

      How can teachers game the system by giving higher grades? I thought college entrance in America was decided by the SAT, motivation letter, etc… not school grades.

    • What causes 60% of the 70% of students who go to college to never finish in a timely manner?

      If “in a timely manner” means in four years, then quite a lot of them are probably people who take a year off, or who lose some credits while transferring and so have to stay for a fifth year, or … .

      The more interesting figure would be how many never graduate.

      • faoiseam says:

        The usual measure for timely manner is 6 years. See for example Understanding college graduation rate and retention rate when comparing colleges

        Graduation rates are actually reported 2 ways, as the percentage of full-time students who graduate in 4 years and as the percentage who graduate in 6 years. The percentage of students who graduate in 6 years is a federal government reporting requirement. Any school that awards federal student aid dollars (which is just about all of them), is required by the Student Right to Know Act, to supply the federal government with certain information, including the percentage of students who graduate within 150% of the typical time to a degree (or 6 years at a 4 year college). In 2012, the national average for full-time students at 4-year degree-granting institutions was 59 percent. So, 59% of students graduated in 6 years, nationally.

        So only 41% who go to 4 years colleges fail to graduate in a timely manner.

  15. OriginalSeeing says:

    In Texas if you graduate in the top 10% of your senior high school class, then you are automatically accepted into a Texas public university of your choice. That includes UT Austin and Texas A&M.
    https://en.m.wikipedia.org/wiki/Texas_House_Bill_588

    There’s also a separate law that gives full freshman year tuition to anyone who graduates their high school as a valedictorian.

    This doesn’t necessarily mean that you will get into the specific college of your choice inside the university, but it does mean that some of the best colleges (like UT Austin’s Business College) are filled to the brim with valedictorians and are exponentially difficult to get in to if you aren’t a valedictorian.

    This was the case for UT Austin’s business college around 2005, but it wasn’t the case for their engineering or STEM colleges at the time. I don’t know what things are like today. I got automatically accepted to UT Austin by being in the top 10% of my class and into the engineering department because of my SATs, but I found it very unfair and un-meritocratic to have the business school flooded by valedictorians. The high school I went to was full of extremely smart students who did extremely well on their SATs, but only one of us got to get the valedictorian title and the rest of us were essentially punished for being at a school full of smart students instead of one where the competition was drastically weaker. (This also led to one of my friends leaving to a different school for her senior year just so that she could steal the title of valedictorian there and get free tuition.)

  16. gbdub says:

    No offense to Canada, but in terms of both reputation, school type, and size, their top 3 are probably more comparable to the top US publics. For whatever the global us news rankings are worth, McGill is at 43, behind most of the UCs, Washington, Michigan, UNC, Minnesota, and Wisconsin (and just ahead of Ohio State, which is absolutely huge with 45k undergrads). Start adding up all of those students and it starts to look a little better for the US, though I think it’s probably still fair to say there are more “elite seats per student” in Canada.

    (The global US News rankings are a little weird – they are oddly inconsistent in a couple places with the US News “top US public universities” list I cited in my previous comment. Notably U of Washington is much higher and U of Virginia is much lower – they rank Washington as the 10th best “global university” but it’s all the way down at #59 on the list of “national universities”!).

    • rlms says:

      On the other hand, if I were looking for employees I would give people going to a university that is top in their country but not in the world a prestige boost to account for people tending not to move.

      • gbdub says:

        Sure, but you could say the same for US States. I wouldn’t knock a good candidate who lives in Texas for going to UT Austin instead of UCLA.

        How are the Canadian public universities organized? Do they cost the same / have the same acceptance rate for people all across the country, or are they provincial?

        • rlms says:

          Definitely.

          No idea, I’m not Canadian!

        • Procrastinating Prepper says:

          They are provincial. In-province students pay the cheapest price, out-of-province a little more and international a lot more. Even more so than in the US, Canadian universities have been raising the share of international students to make up for the public funding not keeping up with inflation (the three unis above have 20-25% international student enrolment).

          I think the biggest difference, though, is that our cost disease has not advanced as far as the US. Even an international student starting now would only pay between $30,000-$50,000 for an entire degree at a top public uni, so we can draw from a much broader population of international students. And since Canadians are mostly competing with other Canadians except for the most ambitious, we strictly benefit from our top unis being big public ones, with no Ivy-equivalents to compare ourselves to.

          • johan_larson says:

            At least the two universities I checked (Waterloo and Toronto) only have two price tiers depending on student origin: domestic and international. Tuition varies quite a bit depending on the subject studied, though.

            https://uwaterloo.ca/future-students/financing/tuition

          • Aqua says:

            It varies a lot by subject. Eng at Waterloo was 65k-ish total (in province). Art would be around half

          • anonymousskimmer says:

            @Aqua
            First time I read that I thought you were saying that “English” cost 2x more than Art.

            Premature truncations – the bane of scientists and everyone else.

  17. naath says:

    Cambridge, UK, is ~3500 per year for a total of presumably around 12,000 (most degrees are 3years although some are much longer) undergrads. Here the pressure against “take more” is at least in part “physical space”; is there a similar issue at Harvard? Also small-group tuition with 1 eminent professor and 3 undergrads relies on a ratio of professors to undergrads, and if you have more students you get less teaching time each, or more (and thus less eminent) professors (since the world contains only so many amazing people in any field)

    • rlms says:

      I think Cambridge is actually constrained by quality of students for many subjects (in some years they won’t fill courses to lecture theatre capacity). A lot of supervisions are given by grad students (of whom there is a large number) and most colleges have more than enough money to build more accomodation.

  18. blankmisgivings101 says:

    Has Scott ever engaged with Peter Turchin? I know he has worked on “elite overproduction” as a factor in civilizational collapse using some sort of massive data set. http://peterturchin.com/cliodynamica/elite-overproduction-brings-disorder/

  19. Alex Zavoluk says:

    Why not use APs for admissions? Well, colleges do look at your Junior year AP scores, and the AP classes you take in Senior Year. Waiting until end of Senior Year would make things more stressful (usually seniors slack off a bit after applications are due around New Years, and they would have less time to decide where they’re going and then work out all the logistics). But, more importantly, the obvious Campbell’s Law problems that afflict extracurriculars, SAT scores, GPA, etc. will also affect APs, and to some extent already have. That’s what’s been causing the number of AP tests taken per student to increase so much!

    Among the most prominent colleges, Princeton is the only one over the last generation to have actually succeeded in boosting enrollment (and that by only about ten percent) after it opened the Whitman residential college in 2007.

    Does UChicago count? It was about 4800 undergraduates when I applied in 2009 and approaching 6600 now. Of course, the whole process makes me understand why some colleges would be reluctant to do so. It’s very expensive, involving building several new dorms, and the logistics have been a tremendous PITA for a lot of students. In order to maintain a low teacher:student ratio (another metric used by US News and other outlets, as well as the college’s own promotional material), they have to hire more professors and grad students, and I think build more classroom space. Many undergrads and alumni think that the new application process (making use of the common app) and expanded student population means that a lot of what once made the school special has been lost.

  20. JohnNV says:

    Possibly relevant: Here’s the letter I wrote to my high school earlier this year when they asked if I would recommend them to friends. (A top private high school in a medium-sized southern US city)

    “Although I valued my time at X and the quality of education I received there, I’d find it hard to recommend attending the school now to anybody but the richest of my acquaintances. In my graduation year of 1995, tuition was $4800, 14% of the median US income that year of $33,600. That’s a stretch for a middle class family, but possible. Today, X charges $25,000 for tuition with a median US income of $61,000, a full 41%. That puts it out of reach of me and virtually everybody I know.”

  21. iansimon says:

    I don’t know if this is really true or how exactly to measure it, but my sense is that over the past several decades the returns to social status have increased. I also don’t have a good explanation for why this would be the case, but I will vaguely gesture at the decline of religious practice.

    • HowardHolmes says:

      Maybe so since religious practice is all about status. People have to get their status from somewhere so if less get it from religion they have to find other outlets. I do not think the overall interest in status has changed since over 99% of humans have always spent the bulk of time and energy to achieve status. Commenting on this blog is all about status (including this comment).

  22. thenuttynetter says:

    So at some point, you mention that a skilled professional from 1800-1900 would have made about 3-5x as much as an unskilled laborer and 3-4x as much as a clerk or a teacher.

    However I think today that the ratio is much much larger. The US minimum wage is 7.25 USD/hour or 15k USD / year. I could also see it going up to 20-30 USD/hour for someone with more skills and experience. I think a lot of low skill labor is pretty close to that, even with years of experience. A teacher might make between 30k-80k (it looks like ~100k happens but is on the very high end).

    On the other hand, an engineer at a top tech company could easily start at 150k-200k / year and 300k-400k is not uncommon (https://www.levels.fyi/).

    So these days, I would say that a skilled professional is closer to earning 4x-30x what an unskilled laborer makes.

    To get the 3x-4x type numbers, you need the skilled professional to only make 150k and the unskilled laborer to make 50k, which seems like it’s pushing the numbers.

    • Mustard Tiger says:

      There’s still some variation in “skilled professional,” though, and SWEs are on the high end. Other types of engineers (mechanical, electrical, aero, etc.) aren’t starting at $100k, and $150k would be a nice career goal after years of experience.

    • Alex Zavoluk says:

      Maybe it depends on the profession. Recall Scott’s post from a few years ago: https://slatestarcodex.com/2015/07/28/non-dual-awareness/

      Most lawyers seem to make less than the mean of 80K/year, with a few on the tail making 165K (looks like 1-2 years out of law school). But the bulk make well under 150, and in some cases make comparable salaries to laborers and less than teachers.

      • thenuttynetter says:

        @mustard and @zavoluk: I mostly agree, although I wonder if that goes even further to explain why the top colleges are so much more competitive. If the job opportunities which are the very best become even better, and a top college helps one to get those jobs, it makes sense (from an expected value point of view) to work even harder to get into an elite college.

  23. thoramboinensis says:

    Not seeing this elsewhere: part of the decrease in acceptance rate at the most selective schools is an increase in international applicants. Though the number of these applicants has gone down nationwide in the US, it’s gone up at Harvard (and I imagine the other blue-chip universities). Source.

    Scott notes that the international student percentage at Harvard has only ticked up a tiny bit. That’s not really a surprise though, since there’s almost certainly a quota for int’l students. (Aside: even when I was an undergrad there a decade ago, I remember thinking the average non-recruited athlete int’l student was a notch above the average non-recruited + non-legacy American student in aptitude.)

    It’s kind of a genius way to game the ranking system actually. Have a quota no one objects to (for int’l students); send your admissions officers on tours of Africa, Europe, Asia to gin up applicants (I’ve heard from an admissions officer that the int’l applications they receive are very closely tied to this trips); decline an ever larger number of those applicants; rise in the college rankings.

  24. Reasoner says:

    So what should the younger members of our audience do about this situation?

    In California, if you’re a high school student, I think you can take the CHSPE test and start attending community college when you’re 16. This saves money, time, stress, and it’s easier to get into a UC as a community college transfer than as a high school graduate.

    There’s this blog post from academic success guru Cal Newport, on how to hack impressiveness. Newport mostly discusses this in the context of hacking impressiveness so you can get into a top university. But you can apply the same thinking to hacking your resume. If a high schooler took all the time, money, and effort they were planning to put into fitting the mold of an ideal college applicant, and instead spent all that time, money, and effort transcending the mold of an ideal employee (maybe with some networking to get past HR screens), where would that leave them?

    For the budding software developers amongst us, TripleByte (SSC sidebar advertiser) doesn’t care if you went to school. Here’s a recent blog post about a teenager who used their service successfully.

    Another hack is to be from a country which isn’t very populous and participate in international competitions like the IMO. Then you can put on your application that you were a gold medalist (in a principality filled with future subsistence farmers but who’s checking?), which sounds more impressive than getting the thousandth best score in a country with millions of bright people.

    You can focus on a master’s degree at a prestigious institution. Master’s degree programs are seen as cash cows and I think they’re often easier to get into than undergrad programs. In the tech industry, some recruiters say a master’s is actually less impressive than a bachelor’s, but presumably, not every recruiter in every industry has caught on. Classic post on this. Another old post I found while searching for that one.

    • Reasoner says:

      I just remembered Cognito Mentoring, a rationalist website about life optimization for bright teenagers. It’s from 2014, so some advice might be a little dated I guess. I have no idea how fast the state of the art moves in this area.

      Does anyone have any other advice for teenagers? I’ll bet there are a lot of them reading this thread hungrily.

    • sharper13 says:

      If you’re smart, you can take and pass the CHSPE when you’re much younger than 16, you just need cooperation from your school administrator. If your parents file a free private school affidavit in order to home school you, they become your school administrator and can list you as having completed the 10th grade and thus qualify. So you can effectively start college in CA as soon as you’re ready.

      Note, this doesn’t work as well for initial college outside CA, because those schools aren’t required to accept the CHSPE on an even-to-HS-diploma basis, so YMMV. It works really well to get a quick 2-year degree at an inexpensive CA JC and then transfer. If you’re smart and can carry a 4.0 at said CA JC, then not only can you transfer almost anywhere, but you’re also more likely to pick up a merit-based scholarship for transfer students.

      Of course, if you pass the CHSPE and start college when you’re 13, try and resist the temptation to return to H.S. for a couple of years just for the fun of it, because while they are required by law to let you in if you’re under 18, and it’s enjoyable to not have to worry about things like grades and homework in H.S., that will still set your master plan back a couple of years. Just don’t ask me why I took AP US History after already passing two semesters of US History in College. The AP test does still get you some elective credit, though.

      • BBA says:

        At the age of 13, I got a high school diploma from the tiny private school I’d been attending. And I do mean tiny – its graduating classes were often in the single digits. I didn’t see the diploma as proof that I was ready for college, because emotionally I wasn’t. But I had exhausted all the school’s intellectual resources, so they sent me out into the world.

        I attended public high school for a few years afterward, was miserable there (also my family moved in the middle), and ended up enrolling in college at 16. I never really learned how to deal with people. I still can’t. Thankfully I got a high-paying job where I don’t really have to, though after being told what a genius I was for so many years I still feel like a failure because I’m not curing cancer or whatever.

        Anyway, I think if a kid finds standard K-12 schooling remotely tolerable they should stick with it, less for the quality of instruction than for the whole typical social experience that I never got and my brother (for instance) did. Like, it’d be nice to know if I’ve always been a friendless loner because I’ve been an outsider, or if that’s my natural state of being.

        • I also went to college at sixteen, and in my case I think it was the right decision. Like you, I was intellectually precocious and socially retarded. If I had waited until I was eighteen I would still have been socially out of place. By going when I did, I at least got to something close to an appropriate intellectual environment.

          Eugene Volokh graduated from UCLA at fifteen, which should make both of us suitably humble.

        • sharper13 says:

          My experience was that going from Middle School to a JC, at the JC (at the time, this was 30 years ago) the other students actually wanted to be there and if they showed up for class, they wanted to learn. I think I did better hanging around with that maturity level than I would have going to 8th & 9th grade. It helped that I was tall enough early enough (6′ at 13) to pass as a college freshman, so everyone just assumed I was about the same age, I just couldn’t drive and I wasn’t interested in the drinking/partying scene anyway.

          Going back to H.S. for 10th & 11th grades after two years of college, I didn’t have to take the teachers seriously. Don’t get me wrong, I was polite and interested when they had something I wanted to learn, but otherwise just read whatever I wanted in class. They mostly loved me anyway, except I never did homework I wasn’t actually interested in, but passed the tests anyway. After being used to the way more mature kids in college, I mostly just laughed at the H.S. kid’s antics, saw through all the silly status B.S. and hung out with whoever I wanted to, whatever clique they belonged to. It probably helped that I was athletic (lettered in three sports each year), but I was also the editor-in-chief of the school newspaper both years (only one who could use the computer systems to create the paper that well), did stuff like Model UN and Mock Trial, etc… I was there to have fun, which meant I took the extracurricular stuff I wanted to do more seriously than the actual classes. I was never “popular”, too nerdy and uncaring for that, but sort of existed outside what seemed like the usual High School hierarchy.

          Socially, K-12 (and K-8 especially) has always seemed the worst to me, because it’s setup for kids to only socialize with others their same age. As a result, they’re all more or less suffering from the same problems, but don’t have as many good examples around from people who have already learned to deal with them to see what that looks like. Some places, notably K-8 Montessori schools, deal with this better because they combine multiple years with competency-based learning, so those kids get to interact with a wider age range.

          • Steve Sailer says:

            Right, junior colleges aren’t prestigious, but they are decent learning environments because, unlike high schools, they don’t have truant officers rounding up students who cut classes. Everybody in a JuCo classroom is there voluntarily, which makes the class environment better than in high school.

        • Rebecca Friedman says:

          For whatever it’s worth, I went to a tiny private school up to about 13-14 and was homeschooled thereafter. I definitely notice social effects – in college I got on better with professors than fellow students, and I still get on very well with people who are a generation or sometimes two back from me, better than most people my age do, and interact less fluidly with my own generation. (Most of my teenage years were spent hanging out with people 5-30 years older than I was.) That said I have a number of close friends about my own age, and do seem capable of making more. And I spent my teenage years in an environment where bullies could be never seen again and most people who interacted with me liked me (because I was moving in communities where being helpful made me friends) which was really good for self-confidence and finally getting over being shy. IMO my parents made the right choice; I’d have been a weirdo and shy in high school, and that’s much more likely to lead to bad outcomes than what in fact happened.

          (My mother, who has a personality very similar to mine, went through an ordinary school, by what she’s said didn’t really relate to “kids her age” either, and is somewhat more shy as an adult than I am. Which is part of why I suspect I was lucky to miss it – and probably also part of why she let me.)

          I’m really sorry your own experience was not nearly so good. For what it’s worth, I’ve had at least one close friend who did her learning to deal with people as an adult (in her case due to severe autism). You’ve probably heard this already, but hobbies help, especially particularly friendly/low-strung/not-status-focused ones, double particularly when they are Doing Something so that your social skills aren’t the only thing people can see about you.

  25. wwbrannon says:

    I think what people may be missing is that time is a thing:

    In a world where there are 10 places at Harvard every year but 9 applicants, everyone who can read Latin gets in, and no-one bothers to study harder than what it takes them to meet the minimum requirement.

    In a world where there are 10 places at Harvard but 11 applicants, the worst Latin-reader doesn’t get in. But someone who realises they’re the worst Latin readers will practice more, and someone else will be the worst. That person will then practice enough to make someone else be the worse, who then practices a bit more. Given enough time, the standard keeps rising as everybody needs to stay ahead of everybody else; eventually everyone hits the ceiling of Latin literacy and starts taking up the violin, then competes on violin-skills until they all have to start climbing Everest.

    The only equilibrium is where everyone exerts maximum effort on top of their abilities.

    This is only true if attending Harvard is infinitely valuable. It doesn’t make sense to exert more effort to get in than your expected benefit from attending. That is, students are balancing two things:
    o) their marginal cost of college prep: the money and time cost of the marginal hour spent on getting into college, and
    o) their marginal benefit from college prep: the increase in the expected payoff of Harvard, given the chance of not getting in, over the next-best alternative.

    In equilibrium, if everyone is rational, those should be equal and effort should stop short of 24/7/365 college prep.

    So if there’s increasing effort spent on college prep, isn’t it because college is becoming more valuable? (They aren’t making more hours in the day, so the cost of prep can’t be declining much.)

  26. kaminiwa says:

    The comments about preparation-needed-to-qualify might tie in to the general sense that life is more difficult/stressful/unhappy despite a general increase in material goods: more and more of our free time needs to be spent building skills. Children are building extra-curricular activities instead of playing. Employees are studying and doing “passion portfolios” instead of paid apprenticeships. And if you’re slacking off and actually enjoying your free time, you’ll have to deal with the stress of potentially falling behind your co-workers.

    The programming field seems to suffer heavily here: There’s a constant churn in what skills are employable, so staying steadily employed doing the same thing for a decade will leave you at the bottom of the pack when you next have to get a new job. And since the average job only lasts a few years, you’ll be facing the prospect of getting a new job often. For bonus points, the more often you job-hop, the easier it is to find someplace that’s using the latest fad. Work there, get it on your resume, then hop to a new job that pays even more. Use the status from that job to get in to wherever the next fad starts.

    • johan_larson says:

      There’s a constant churn in what skills are employable, so staying steadily employed doing the same thing for a decade will leave you at the bottom of the pack when you next have to get a new job.

      Well, yes, there is some of that. It’s also quite possible to find a job working on a decades-old codebase that is written in plain old C and still needs extensions and upgrades because there’s a user base of millions that depends on it. If you want stability in the software industry, you can find it.

      On the other hand, if you really want an environment of rapid change you can find that too. Web dev in general is very keen on working with the latest shiniest tools. And within web dev, front-end development is the churniest of all.

  27. thetitaniumdragon says:

    I went to Vanderbilt in the early to mid 2000s. I can tell you that even back then, every elite financial institution gave very generous financial support to people. In fact, while interviewing around colleges, they all told me that you shouldn’t worry about the supposed “sticker price” because no one actually paid it due to the generous scholarships. Even people whose families made $100k+/year got significant financial aid. It was actually something of a joke that we would go around looking for the mythical people who actually paid full tuition; I never met one the whole time I was there.

    The idea that these colleges are super expensive simply isn’t true.

    Moreover, the lower admission rate is in large part attributable to people applying to more schools, and I suspect more people applying to schools they’re not really qualified for.

    It’s also worth noting that there’s a ton of very, very good colleges in the US; dozens of them, really. How many good colleges can you name off your head? Probably the entire Ivy League, MIT, Caltech, UC Berkeley, Duke, Vanderbilt, and John Hopkins at least, and probably more if you actually tried. The fact that our top tiers are small individually doesn’t mean we don’t have tons of people going to top tier colleges. Indeed, part of why they have super elite reputations is likely because they’re able to be so selective.

    • AG says:

      And yet, empirically people are getting stuck with student loan enough to significantly lower their buying power. Notably, those who are less well off tend not to have the knowledge to access all that financial aid.
      I got less than 10% aid, and that doesn’t include the expenses of room/board/textbooks, either.

      And again, empirically, if so many people were able to go to good colleges in the US, why would such a cheating scandal emerge? Revealed preferences and all that.

      • And again, empirically, if so many people were able to go to good colleges in the US, why would such a cheating scandal emerge?

        Probably because very rich people sometimes have children who are either insufficiently able intellectually to be accepted at what they view as an elite college or too focused on things other than academics.

    • Edward Scizorhands says:

      It is not generous support. It is price discrimination. They will look through your tax forms and decide exactly how much you can afford. Only the mob is more intrusive, and I’m not even sure about that.

  28. onyomi says:

    Assuming there is some truth to the idea of consulting as selling prestige (I see some pushing back on that, at least wrt specific fields below; don’t personally know enough about it to have a strong opinion) and related to bureaucracy as active ingredient, it strikes me that the function “prestige” frequently serves in society is, like bureaucracy, as a reserve of ass-covering power.

    Basically, no one wants to be directly responsible for when things go badly, so they look for ways to do what they think is best within the constraints of maintaining the appearance of due diligence so they can’t be taken to task too badly if things go south. If prestigious consultants with Harvard MBAs sign off on your business decision and it fails well then that looks a lot better than if you just made that decision on your own. Meanwhile, Harvard, as a kind of giant well of accumulated credibility (yes I know that has taken a hit in some quarters recently), can afford the hit if it turns out their MBAs gave bad advice because of centuries of accumulated prestige and, presumably, a continued ability to credential others who give good advice.

    • The Nybbler says:

      Indeed, though even giant wells of accumulated credibility can run dry. A Definite Beta Guy talked about a good path to executive levels being to join a Big 4 audit firm. Well, the Big 4 has changed in number from time to time, mostly due to M&A but most recently because Big 5 firm Arthur Andersen burned _all_ of its giant well of credibility on Enron. I don’t know what happened to the junior auditors who had the bad luck to join that firm rather than one of the others, but I doubt many of them are CFOs now.

      • Education Hero says:

        I’ve met a number of senior managers and partners at my Big 4 firm who once worked for one division or another of Arthur Andersen that we absorbed after their fall.

        Big 4 and various other firms bought out nearly all of the professional groups that formerly comprised Arthur Andersen. Even the Andersen name lives on through Andersen Tax; that group was bought out by HSBC as Wealth Tax and Advisory Services, and later spun off as Andersen Tax (today they have a significant practice providing tax and other financial services to high-net worth individuals in the Bay Area).

        Many people took financial hits (reduced salaries, lost bonuses, vanished pensions, etc) and disrupted career advancement, but my understanding is that there was no employment bubble because all of Arthur Andersen’s clients still needed services, and there was significant specialized client knowledge baked into the engagement teams that serviced them.

  29. peak.singularity says:

    I’ve seen conflicting data on whether long-term growth has really decreased and when that started, and you can see some discussion of this in the comments. If it has, it would explain a lot, and would mean there’s still room for things to get worse.

    I’m not aware of any conflicting data ?
    The situation is pretty clear by now – our civilization hit its peak in the 1970’s, up until circa 2008 we were in the stagnation phase – we basically gave up on the future of our civilization in the 1980’s… (assuming we had any choice in the matter ?)
    And now we’re (likely) in the collapse phase – which is going to last a while, in human terms.

    (Note that this is for the Western world – the Chinese are following a similar trajectory, with some lag – and with the global supply chains, we’re pretty much part of a single super-civilization by now !)

    Also, our civilization is an exceptional one in that we’ve figured out how to tap the vast troves of stored hydrocarbon energy (and other resources) – the next civilizations won’t have this luxury !

    Now, it’s possible that we might yet be able to break the cycle, by giving up (our) humanity, and letting the AI (or some weird “enhanced human”/AI hybrid) to take control… with maybe some hope to expand beyond Earth.
    But you can understand why most people aren’t thrilled by the prospect.

    Myself, I don’t know if the risk is worth it – and if we take it, and fail anyway, having kept promoting growth at all costs, we risk wrecking our environment to the point where humans won’t survive the environmental changes.

    But then, again, this is the only shot at it that we’ll ever have – the hydrocarbon reserves take tens of millions of years to grow back, and we’ll have mined the easily accessible minerals anyway – and the typical mammal species lasts only for a million year or so, AFAIK mostly for genetic reasons.

    • The Nybbler says:

      Western Civilization hit its peak…. prior to the microcomputer boom, let alone the internet boom, and 20 years before the fall of the Soviet Union?

      Pull the other one, it’s got bells on.

      • peak.singularity says:

        Ah, yes, I was afraid of a comment like this (but not as much as being misinterpreted the other way).
        If you want to look it in another way, the increase of a slope has to slow down before hitting the peak.
        In this sense, one could say that the peak “started” in the 1970’s, and was reached circa 2008.

        • The Nybbler says:

          Then you are claiming the 1970s were a negative inflection point, not a peak. I still think it’s silly, for the reasons I stated.

          • peak.singularity says:

            Again, I’m surprised that this would be considered a controversial claim – I pretty much assumed that this was common knowledge, especially here !
            Yes, the data is noisy, but isn’t the overall trend pretty clear ?

          • Ghatanathoah says:

            @peak.singularity

            I’ve made a study of the 70s recently. It looks to me like the 70s were some kind of dip or slump, not a peak or inflection point. In the 80s our civilization pulled itself out of it and reached new peaks. The 80s weren’t us giving up on the future of our civilization, on the contrary, they were our civilization getting its act back together.

            You can really see this with 70s futurism and science-fiction. It was full of decayed future societies, global ecological collapse, overpopulation, etc. These societies were often forecast as only decades away! Those predictions didn’t come true, of course, humanity rallied, crime fell, markets grew, communism collapsed, and we had the Green Revolution.

            I have noticed some parallels between this decade and the 70s. In particular the some parts of the system feel broken and decayed, and we’ve lost trust in some parts of it. but we fixed the problems of the 70s. We will fix these ones too. Objectively they are smaller than the problems of 45 years ago.

            “Soylent Green” takes place in 2022. We aren’t there, not even close. And it’s unlikely we ever will be.

          • peak.singularity says:

            @Ghatanathoah

            But this is actually exactly what I’m talking about when I say that “we basically gave up on the future of our civilization in the 1980’s”.

            That’s when Reagan got elected and scrapped most of the environmental policy that Carter started to put in place, symbolized by the removal of solar panels from the White House.

            Sure, it *feels* like “we got our act back together”, because we decided to pursue the growth at all costs strategy which Sam Altman is talking about.

            Except this is a short-term strategy. It requires exponentially increasing amounts of power (and the resulting entropy, dissipated in the environment).
            It increases the complexity, and therefore the maintenance costs of the dissipative system :
            http://www.francois-roddier.fr/blog_en/?p=45

            The 70’s themselves were a consequence of a shock happening because the USA ran into the power supply issue : in that case they have reached the maximum production of (conventional) oil.

            We were smart enough to recognize (and even predict) the issue, but not strong enough to be able to face the consequences, and change our behavior, and instead decided to kick the can down the road for a bit more (and start a few wars in the Middle East to keep our oil supply secure).

            Fast forward three decades, and this time we’ve reached peak *world* production of (conventional) oil – this popped the subprime bubble, and caused the 2008 recession.

            We’ve decided to double down *again*, and keep growing with the help of tight oil and bituminous sands (note that this has catastrophic consequences if you believe in most versions of global warming).
            For how long this time ?
            Laherrere, who predicted the peak world conventional oil (for 2010 instead of the real 2006), thinks peak world unconventional + conventional oil will happen in 2019.
            (Middle East is always an uncertainty of course.)
            This should shortly cause another recession. (Some economists are already “sensing” it coming…)

            What will do next ?
            There are a few potential candidates to keep kicking the can : shale oil (aka kerogen, aka “uncooked oil”) and methane clathrates – both still considered uneconomical for now.
            Arctic oil – last attempts have mostly failed, global warming is likely to make it easier though.

            Solar power – would require decades of investment in a completely different kind of infrastructure, WHICH WE SHOULD HAVE KEPT DOING WHILE WE STILL HAD TIME TO, STARTING WITH THE 70’s ! (ahem, sorry)

            Various forms of nuclear (which was what King Hubbert was hoping for when he predicted the 70’s peak oil in 1956) – we actually DID build a large fleet of nuclear reactors in the 70’s ! But it pales compared to our global demand, has the same issue as solar that it doesn’t produce the kind of energy that we’re used to for powering our transportation – and it would seem that we don’t really know how to do it safely, *especially* in times of crisis when it’s tempting to skimp on safety.
            (It’s a shame though that we didn’t spend more energy on various forms of nuclear prototypes, like thorium…)

            The Green Revolution “ended” in the 70’s.
            It’s part of the problem, as it requires a supply of oil-based chemicals, and mining for minerals some which we might currently only have a few decades worth of if growing the extraction speed exponentially – Phosphorus seems to be especially problematic.

            Again, “decayed future societies, global ecological collapse, overpopulation” seems to me to be pretty much where we are NOW – collapse is a phase, not a single occurrence, and takes a while in human terms (it took centuries for the Roman Empire). It is also unevenly spread, both temporally and geographically, and is not about going down monotonically, with occasional bouncing back. (The Soviet Union lost the competition, and got there early.)

            Soylent Green is “unlikely” because overpopulation is a short-time situation, quickly “fixed” by revolution, war and/or disease – we have recent real-world examples in the form of the Arab Spring (Egypt, especially) and Syria.

            I’m surprised that you think that our issues are smaller than in the 1970’s ?

          • I don’t think culture war topics are appropriate in this thread under current rules, so I will just note that I strongly disagree with the factual assumptions underlying your post about why things are on their way down. I’ll be happy to discuss the question further in a suitable open thread.

          • peak.singularity says:

            @DavidFriedman
            Uh, I fail to see any link to culture :??

            But yeah, the most certainly requires its own thread, even if it’s Scott himself that brought the topic up…

          • @peak.singularity:

            “Culture war” here is a term of art. It means issues that people with differing political views tend to argue over, often passionately. Climate change, overpopulation, and related issues fit that pattern. As do abortion, gun control, and immigration.

          • peak.singularity says:

            @DavidFriedman

            Ah, well, I’ve *tried* to keep climate change and overpopulation topics as secondary to the GDP-energy link, but they do keep creeping up…

  30. synaxarion says:

    Obviously my own experience is anecdotal, but as a senior at a competitive public high school in the Bay Area…I got the impression that basically nobody, and especially nobody over the age of 30 or so, had any clue what they were talking about.

    From what I can tell, admissions have become more competitive because a small proportion of students have become very, very good at identifying how to demonstrate extreme academic potential. One person suggested that colleges use AP tests to filter for academic success–AP tests are laughably easy. I’ve gotten eight 5s and am registered for five more AP tests this year. If I thought the number of AP tests I took might have any impact whatsoever on getting into college, I’m fairly sure I could have scored at least 3 more 5s (I have friends who got 5’s on tests like APES and microeconomics after a few hours of self-studying). As it is, colleges are fully aware that the majority of AP tests are completely meaningless and accordingly don’t award credit.

    Instead, look up finalists in academic competitions associated with computer science, physics, and biology (math is probably the best-known, and chemistry’s also pretty big, but lists of campers aren’t as easily accessible), attendees of prominent research summer camps for rising seniors, winners of research competitions; I’d guess that at least 70% of these people go on to undergrad in one of HYPSM. That’s way better than random chance! And this is relatively well-known, at least where I’m from. But success in these contests is pretty difficult, and increasingly so for the more established contests–there’s a significant contingent of kids who start preparing for math competitions starting as early as elementary school. So within this group, things are already super competitive, since a lot more people are trying to win finalist standing or spots in summer camps than are available. Incidentally, at least for academic competitions and research, these kids are overwhelmingly Asian or white and come from just a few areas.

    There’s similar competition over music, and chess, and sports (which leads into the entire sports recruit business). Add in legacies, and probably half of the class is filled already, yet looks pretty homogenous. Colleges also need to keep diversity in mind, so at this point there’s very, very few spots left at top colleges for people coming from a traditional upper-class white or Asian background who are just doing traditional extracurriculars (along the lines of school-level leadership/volunteering) who need to distinguish themselves through essays, and I think it makes sense why competition is so intense.

    re UC’s: They’re typically considered better than any other states’ colleges, so it doesn’t really make sense to compare them with any other state school. In particular, I know people who have chosen UC Berkeley’s CS program over CMU’s, though CMU is considered to basically be top of the line CS education for industry. Furthermore, applying to more UC’s is really really easy–once you’ve already filled out an application to one UC, all you have to do is check the box for more! Also, state universities are basically the cheapest option for in-state students (at least compared to out-of-state public universities and private universities), and at least personally my impression has been that the quality of the best state school you can get into will probably approximate the quality of the best private university you can get into, but for half the price. But since quality matches up between state/private schools now, UC’s can’t really function as a safety the way that most state schools do (where you’re guaranteed a reasonably good college education as long as you did okay in high school), since now the only people who can get into UC’s are people who could have gotten into decent private universities anyway. Furthermore, CA is really big, so if you live in the Bay Area and go to UCLA you’re about as far away from home as someone on the east coast going to school in a different state; this means that independence from parents doesn’t really apply to CA students as a reason not to go to a state school.

    • faoiseam says:

      I got the impression that basically nobody, and especially nobody over the age of 30 or so, had any clue what they were talking about.

      Is this in relation to college admissions, or about things in general. If the latter, this feeling was widely shared in earlier days. “Don’t trust anyone over 30”

      One person suggested that colleges use AP tests to filter for academic success–AP tests are laughably easy. I’ve gotten eight 5s and am registered for five more AP tests this year.

      35k students get 8 4s on their APs or better. They do not publish the data more finely grained than that, so I can’t say exactly how many people get 8 5s, but, 4’5 and 5’s outside BC calc are evenly distributed, so a reasonable estimate is that each of the 256 outcomes are equally likely. This would suggest 120 people get 8 5s. I would guess that more than that get those results, but not much more.

      As a Bay Area Senior, you are in a group that tends to score far above what everyone else in the country does, which tends to bias your judgement. When you get to college, you will be horrified by how stupid everyone is (or almost everyone, depending on where you land.)

      I assume you applied to UCs, and I notice that very few Bay Area students get into UCSD, so I wonder if you did. The data, and anecdotes, suggest there is a bias at UCSD against Bay Area students.

      For undergraduate, most CS professors would recommend Berkeley over CMU, thought for grad school, they are much closer. Even then, CMU is definitely not significantly better than Berkeley.

      You suggest looking at winners of competitions, and link to a few. Take “On-demand electrically controlled drug release from resorbable nanocomposite films”, whose second author is senior (I presume). The first author just finished their PhD, and I find work like that a little implausible for a high schooler. I have to believe that, in most cases, work like this is a result of parents pulling strings, as opposed to actual talent. I pulled this one at random, so I apologize if this is you, or someone who actually did the work.

      I have judged some science competitions, and the process left me a little jaded, as the students seemed to have a very shallow knowledge of the work they had done, with notable exceptions. The notable exceptions, the ones who seemed they actually had done the work themselves, were not the winners, sadly.

      • synaxarion says:

        Is this in relation to college admissions, or about things in general.

        College admissions specifically, sorry.

        35k students get 8 4s on their APs or better. They do not publish the data more finely grained than that, so I can’t say exactly how many people get 8 5s, but, 4’5 and 5’s outside BC calc are evenly distributed, so a reasonable estimate is that each of the 256 outcomes are equally likely. This would suggest 120 people get 8 5s. I would guess that more than that get those results, but not much more.

        I’m fairly confident that each of the 256 outcomes is not equally likely; I think performance of AP tests is correlated and the people getting 5’s on some AP tests tend to also be the people getting 5’s on others. I’m pretty sure I’ve personally met over 100 people who got 8 5s.

        As a Bay Area Senior, you are in a group that tends to score far above what everyone else in the country does, which tends to bias your judgement. When you get to college, you will be horrified by how stupid everyone is (or almost everyone, depending on where you land.

        I’ve definitely heard this before, and I think there’s some truth in it! But Bay Area kids also tend to go to top colleges at a rate much higher than most, which I think supports what I’m saying about how a small group of people are basically just competing on a higher level and leaving very few spots for others.

        I notice that very few Bay Area students get into UCSD, so I wonder if you did. The data, and anecdotes, suggest there is a bias at UCSD against Bay Area students.

        I did, and I don’t think there’s any such bias at all. UCSD seems to actually be really straightforward in terms of admissions–based on Naviance statistics, everyone from my high school above a threshold GPA and SAT/ACT score gets into UCSD.

        For undergraduate, most CS professors would recommend Berkeley over CMU, thought for grad school, they are much closer. Even then, CMU is definitely not significantly better than Berkeley.

        Okay, I think based on rankings/talking to my peers that was not my impression (my impression is that CMU approximates Berkeley), but my own argument was also that CMU was not significantly better than Berkeley, which is why UC quality of education is high and why UC’s are so especially competitive; sorry if it didn’t come across that way.

        …the process left me a little jaded, as the students seemed to have a very shallow knowledge of the work they had done, with notable exceptions.

        Honestly, I don’t disagree with this; I think there’s way too much parental involvement, again why the people participating in these competitions tend to be predominantly Asian. However, this doesn’t respond to my point; I looked up the student associated with the article you mentioned, and he’s currently going to Stanford.

        • faoiseam says:

          I’m fairly confident that each of the 256 outcomes is not equally likely; I think performance of AP tests is correlated and the people getting 5’s on some AP tests tend to also be the people getting 5’s on others. I’m pretty sure I’ve personally met over 100 people who got 8 5s.

          I presume there is some correlation. I wish there was a better way to tell how many people got 8 5s. Like you, I know quite a few people who have 8 5s, but definitely not 100.

          The college board gives out data on AP Scholars, with Honors, and Distinction, which gives a little better sense of correlations. It does not suggest that the 5s are all concentrated in a small group. In addition, the number of 5s in each subject is about 20%, despite huge differences in the number of people taking each subject, again pointing against a strong correlation. To get to 8 APs you are taking both Math and Humanities APs, and we know from SAT scores that there is not a huge correlation between the very top scores. For example, only 500 people had a perfect score on both in 2016.

          UCSD seems to actually be really straightforward in terms of admissions–based on Naviance statistics, everyone from my high school above a threshold GPA and SAT/ACT score gets into UCSD.

          I notice that Lynbrook, to take an nearby example, has only 10% of people accepted to UCSD, which seems low to me. I can’t imagine what the GPA and SAT cutoff is that only admits 10% of Lynbrook.

          based on rankings/talking to my peers that was not my impression

          I find it interesting that children would have a different view than professors. I wonder where the attitude comes from. There are a lot of CMUers in the Bay Area, but not that many. My best guess is that the anti-Berkeley thing is a carry over from the rivalry with Stanford.

          • synaxarion says:

            The college board gives out data on AP Scholars, with Honors, and Distinction, which gives a little better sense of correlations.

            Would you mind providing a link?

            I notice that Lynbrook, to take an nearby example, has only 10% of people accepted to UCSD, which seems low to me. I can’t imagine what the GPA and SAT cutoff is that only admits 10% of Lynbrook.

            I think around 1570+ SAT, 3.85+ unweighted GPA is really safe?

          • faoiseam says:

            The definitions of AP scholars are here, but I would guess you them.

            The data as far back as 1997 is here. The Scholar data is a xls file, named “AP Scholar Data 2016.xlsx”. It gives the number of each award by state. 14 ninth graders had 8 4s or better in 2016. Arizona has a lot because of BASIS I guess.

      • Vosmyorka says:

        My personal experience, having attended a nice but not incredibly nice public school, is that your figure of only 120 people receiving 8 5s on their APs has to be very wrong, to the point that a non-negligible fraction of the 35k people in the country who received 8 4s or a better result in fact received 8 5s. My high school had 4 such individuals, which suggests to me a figure in the thousands nationally per year.

        • johan_larson says:

          Ultimately it wouldn’t be particularly difficult to set up a written test of academic ability that ranged well beyond high school work, to the point that pretty much no high school student got a perfect score on it. Or maybe there could be a couple of tests, one for math and science and one for humanities. It ultimately doesn’t matter. The fact that this hasn’t been done suggests that what these elite institutions are looking for isn’t mere nose-to-the-grindstone ability to master academic material. They want something else.

          • What I have long argued for is an essay test, taken under circumstances where it is clear the applicant wrote the essay without assistance. For a student who visits the college, it’s easy. Put him in a room with a word processor and a list of topics to choose from, give him an hour. For students who don’t visit, set up the equivalent near him with the aid of an alum.

            The ability to write is an important skill for college that present approaches to admissions don’t test for.

          • faoiseam says:

            Colleges are now asking for a graded essay written for a teacher, with the teachers comments. The idea here is that it has to be your own work, as the teacher could tell otherwise. I think this is a noble idea, but it misses the point that many essays, at least in the Bay Area, are written by high school teachers themselves, moonlighting as tutors.

            Some AP exams have essays, but I don’t think they are accessible to the college. The SAT essay is actually accessible, you can actually see what the student wrote, but alas, the grading and prompts are stupid.

            The single biggest problem with the current system of application essays is that they are all written with much outside help, so are not probative. GPA suffers from a lack of comparability. The SAT is far too simple, so there really isn’t any common metric by which to grade students, that cannot be gamed. As a result, colleges are full of those children that are most willing to cheat. You get what you measure.

    • Anthony says:

      The UC campuses vary in quality and prestige, and that variation is somewhat dependent on your major. (Back when I was looking, UCSD was higher-ranked in CS than in a lot of other fields.) So almost nobody can use Berkeley or UCLA as their safety school, but many can use Irvine or Merced.

      When I was doing college tours in the mid-80s, the guy at UC Davis told me the top 5 reasons people gave for choosing Davis were:
      1. Very good reputation
      2. Very good to excellent reputation in their particular field
      3. Only, or one of a few, schools in the field (the viticulture major was already established then
      4. Close to home
      5. Far away from home

      Everyone laughed at #5, but I doubt that many were surprised.

    • Steve Sailer says:

      AP tests tend to have the bone structure of a very good test, but are limited by easy grading.

      Advanced Placement tests are scored on 1 to 5, with a 5 supposed to stand for an A in an average 101-102 intro course at an average college, a 2 for a B, and so on.

      Last I checked, MIT only gives credit for 5s, and Caltech doesn’t give credit for any APs because none of its courses are at the level of 101s at normal colleges.

      As tests are switched over to being given on computers with adaptive questioning (get X number in a row right, they start giving you harder questions), I think AP test scoring should be extended from a top score of 5 to a top score of 7. For example, about a decade ago, you only needed to get a little under 60% right on the AP Chemistry test to get a 5. It would make all the sense in the world to give out 6s and 7s that would deservedly get the attention of Admissions departments.

      Like I said earlier, since test prep seems inevitable, we might as well encourage students to test prep on real subjects like chemistry and American history where they might even learn something.

      • faoiseam says:

        AP tests tend to have the bone structure of a very good test, but are limited by easy grading.

        AP tests are fairly well correlated with the corresponding college courses. It is hard to change them, as all colleges have standardized on what Calculus 1 and 2 are. Caltech may be beyond AP Calc, but few other colleges are. Only 25% of Harvard has taken a Calculus course beyond Calc BC, and 35% have taken AB or less.

        I estimate that the very best colleges could admit just on AP scores, with sufficient discrimination to determine who the very best 250 kids are with 8 AP exams. There are 37k students who get 8 4s, (AP National Scholars) so if all the 256 possibilities are equally likely, then there should be 120 kids with 8 5s, and 1000 kids with 7 5s and one 4.

        More precision at the top end of exams does not really make sense, as an A in Calculus 1 is an A, and acing the test to 98%+ just shows you are the meticulous sort. A better solution would be to add new AP tests. Physics has Mechanics and E&M which could be extended in to Light and Heat. An exam on Linear Algebra, or multi-variate calculus would be obvious. The same applies to Biology and Chemistry, where there are natural next course that could be tested.

        I suppose it depends on who you think should go to Harvard. If you want the smartest, you would use an IQ test. If you want the most academically prepared, you would count AP and post AP courses. If you want the people who would make the best reseachers, you pretty much need to include the professors in admission. If you want the future influential people, then you need to admit the rich.

        As an aside, why wasn’t Olivia Jade an obvious admit to USC. She built a high tech business in her teens worth millions (a makeup line powered by Instagram). That is more impressive than all but a handful of other teen entrepreneurs. Ask any college girl whether OJ would be an asset to them, or is a role model, or is someone who they would like to know. The bias against her is just jealous from a certain sort. Why the admission office would not immediately recognize that she was a unbelievably high achiever in the modern world (can you be popular on Instagram) is unclear to me.

        • edmundgennings says:

          I my experience the gap between 4 and 5 on an AP test is pretty large. At the top there are only three possible grades. 5,4, or if one really messes up 3. Some sort of 4.5 and 3.5 would make AP tests more able to distinguish differing degrees of competence.

          • faoiseam says:

            There is not much call for a 4.5 in college admissions. As I mentioned, 37k people get 8 4s. A 4.5 would not help in distinguishing who should get into elite colleges, who would be asking for almost all 5s. For colleges like UCs, a mix of 4s and 5s should be enough.

            I agree with Sailor, that the place more discrimination is needed is at the top end, rather than in the middle. I just don’t think that a 6 iw merited in most subjects, as in the ones I am familiar with, a 5 means that you have pretty much nailed the material, and a 6 would just mean that you hit perfection, either as a result of luck or general fastidiousness.

        • johan_larson says:

          It’s sort of like that saying about banks: you get a loan by proving you don’t need it. Similarly, you get into an elite college by showing you are already doing work well beyond high school.

          Dear Stanford,

          Thank you for your interest in my future career and for the application materials you enclosed. Unfortunately I am unlikely to have time to submit an application. My colleagues and I have been working on a computer security server, and with the prototype up and running, our applications to YCombinator and a handful of other accelerators are currently in process. Assuming one of them is successful, which seems likely given the market we have identified and our early success in serving it, our careers will already have taken flight, and we will have no need for the sort of launch pad you provide.

          Good luck wit your search for candidates. We’ve heard very good things about your program.

          Yours sincerely,
          Able Talent

        • synaxarion says:

          I agree with Steve Sailer, and want to add that colleges also value diversity and personal character way above any of the characteristics you identified. Furthermore, getting a 6 would probably be closer to an 80% than a 98%–currently, several AP tests only require around a 60-70% for a 5. Again, far more than 1000 kids have gotten 8 5’s, especially if you’re including AP tests taken in senior year, and if colleges made AP tests the metric for admission, I’m sure that number would quickly jump to over 20,000. I’m not sure why you’d expect that colleges are looking for academic preparation in their admits.

          • faoiseam says:

            I’m not sure why you’d expect that colleges are looking for academic preparation in their admits.

            If colleges are not looking for academic preparation, what are they looking for? The other choices are a inchoate leadership potential, which sounds great, but which, if it exists, is probably mild sociopathy. Another choice is raw brilliance, which would be best measured by an IQ test, like the old SAT. A desire for a diverse class cannot be used for selection, as that is a property of the entire class, not one individual.

            I can’t think of a better test that a child should be admitted than a proven ability to perform well in college classes. I could ask for a proven ability to independently do world class research, but the rub here is “independent”. All the children who have publications have a lot of support, and it is unclear in each case how much is their own work. When you get them on campus, you can tell what their abilities are. The question is how to tell in advance. Harder exams make a lot more sense than higher grades on (relatively) easy exams.

          • Steve Sailer says:

            The problem with using AP tests in college admissions is that lots of high schools don’t offer AP courses in all or any of the subjects.

            Last year, 2.8 million individual students took AP tests, which is a lot but there are, what, about 15 million (?) high school students in grades 9 to 12?

            https://secure-media.collegeboard.org/digitalServices/pdf/research/2018/Student-Score-Distributions-2018.pdf

            It would be interesting to know what % of Ivy League students come from high schools without an adequate number of AP courses. Probably not all that many, but it would still be a PR problem for colleges to announce that they are going to emphasize tests that not everybody can study for.

            I don’t know a solution for this problem. Any ideas?

            Another issue is that AP tests are only given in May, so colleges can’t use senior year tests. I think, however, the college board could give, say, a half dozen popular half year tests in early December and have them graded over Christmas break.

          • synaxarion says:

            Elite colleges want alumni that will help them maintain a prestigious reputation & donate. The specific flavor of alumni they’re interested in varies a bit depending on the exact legacy the college wants to convey (for example, Stanford goes for startup culture, while MIT is more interested in pure academic brilliance–just the general sense I’ve gotten), but in any case they want students who will go way beyond the norm, not just conform to established guidelines (which setting a standard based on AP tests literally is).

            In addition, colleges are trying to move away from standardized testing, even the SAT (for example, U Chicago no longer requires SAT/ACT scores), because it’s easier for people from wealthier backgrounds to prep for them, so even doing well on hard exams is essentially based on support. At least with research, students have to get along with a professor well enough that the professor will write a rec letter for them (not saying I personally agree with the importance of research in college admissions).

          • Steve Sailer says:

            That’s why I think extending the AP scoring from 1 to 5 to 1 to 7, using adaptive testing on computers, would be useful in college admissions. Getting a 5 just isn’t that hard on a lot of AP tests: e.g., my son got a 5 on AP Bio in 7th grade.

            With a printed test, the selection of questions has to be the same for each test taker, so they can’t throw too many tough questions at students. But as testing slowly all moves onto computers, they can interactively throw harder and harder questions at students who are acing the easy questions.

          • faoiseam says:

            9 states have laws that all schools must offer at least 4 APs, and about half of states have laws that APs or dual enrollment must be offered. The states with laws are the Southern academically weak states.

            I don’t think there are many schools left where people do not have access to a broad range of APs. Take Salinas High School as an example of a mostly Hispanic, rural area. It offers 18 APs.

            There are 20k schools that offer APs, but 36k schools in total. I can’t find the 16k. Every school I look for has at least 4.

          • faoiseam says:

            Elite colleges want alumni that will help them maintain a prestigious reputation & donate. The specific flavor of alumni they’re interested in varies a bit depending on the exact legacy the college wants to convey (for example, Stanford goes for startup culture, while MIT is more interested in pure academic brilliance–just the general sense I’ve gotten), but in any case they want students who will go way beyond the norm, not just conform to established guidelines (which setting a standard based on AP tests literally is).

            Alumni that will donate are uniformly white and male. The colleges I know have a huge problem in fundraising coming up, as when they moved from primarily male, to co-ed they halved their donor base. All the URMs, and possible most of the Asians admitted will never donate.

            Further, donations come from the least progressive section of the students. People whose college experience was traditional, males on sports teams, member of fraternities, etc. are far more likely to be donors. Progressive people do not give to their alma mater, they tend to donate to causes instead. I can understand this from a progressive point of view. Why give more money to Harvard, when you could fight climate change instead.

            This causes an issue, as the progressive movement over the last 50 years is draining the pool of donors. People give the largest amounts in their 70s, so schools are drawing from people who attended in the 70s. The huge changes in colleges in the 80s, and later, may prove to be the end of large gifts, and hence endowments, as the later classes, that were more diverse, may not have the cohesiveness to donate. The evidence so far suggests that they will not.

          • Why give more money to Harvard, when you could fight climate change instead.

            That attitude isn’t limited to progressives. In my case, why give money to Harvard when I could give it to the Institute for Justice instead?

            I’m not sure what the dividing line in attitudes is, but I don’t think it’s as simple as left/right political views. It might have something to do with rationalism.

          • Steve Sailer says:

            Thanks, faoiseam.

            Are you familiar with any published studies of who the big donors are to colleges? I came to the same conclusions as you from reading articles about individual big donors (e.g., Mr. Dornsife who gave $200 million to USC a few years ago was the shotputter on the 1965 USC national champion track team, his parents were USC grads, and at least one of his children is a USC grad, he made a fortune in heavy industry, etc etc). The plural of anecdote is anecdata and if you read enough anecdotes, they start to add up.

            But I’d be fascinated to see if any of the studies colleges must have done of who their big donors are have been leaked to the public.

          • Edward Scizorhands says:

            In the early 1990s, I was the best student in my fairly well-off high school as far as college prospects went, and got into a HYPSM. (This was unusual. My parents had to yell at my guidance counselor to get him to send colleges my transcripts, and said counselor also didn’t know what achievement tests (now called SAT II or SAT Subject tests) were.) I only had 1 AP class done by junior year, and only got a 3 on it. I had 3 in my senior year and blew people away by getting 5’s on all of them.

            I knew things had gotten rougher, but to hear that I would now need to more than double this is pretty amazing.

            Also, I was above average in how many AP tests I had passed. My college started getting pissy about all the AP credits kids were coming in with around then, too.

        • Steve Sailer says:

          Thanks for doing some thinking about AP tests.

          Good point about Olivia Jade. She sounds like the kind of celebrity that Harvard likes to admit.

      • Steve Sailer says:

        Charles Murray is a big fan of the SAT Subject tests (which used to be called the Achievement Tests and the SAT II), which are like 1 hour versions of the Advanced Placement tests. He thinks a combination of SAT Subject tests does a slightly better job of predicting college GPA than the standard SAT test.

        https://en.wikipedia.org/wiki/SAT_Subject_Tests

        It looks like the more elaborate AP tests have started to pull ahead of the Subject tests in popularity.

        • faoiseam says:

          The number of people taking SAT subject tests is an order of magnitude below the number taking APs. The most popular AP (Lang) has 600k takers, while the most popular SAT 2 (Math 2) has 60k. The next most popular SAT 2s are Chemistry (32k) and US History (23k) which are dwarfed by AP Psych (300k) AP US History (300k) and Calc AB (500k, including BC subscore).

          The big advantage of SAT 2 tests are they allow schools and admission committees to translate GPA into actual achievement, as the level of SAT2s for a school show the level of grade inflation.

      • johan_larson says:

        Like I said earlier, since test prep seems inevitable, we might as well encourage students to test prep on real subjects like chemistry and American history where they might even learn something.

        Yes, it would be useful to push the effort toward learning real material rather than mastering the peculiarities of aptitude tests. Or better still, start doing some actually useful work.

        Over here in software land, I expect a motivated high school student could, with a bit of guidance, learn enough about software development from books and online sources to start contributing to open source projects before they graduate. They would start with really simple stuff, of course, but there are always more bugs to fix and doc pages to update. This would build real expertise and would produce actually useful results.

  31. Jon3 says:

    Why do you think that “colleges are fully aware that the majority of AP tests are completely meaningless and accordingly don’t award credit”? Rice gave my daughter enough credit for her AP tests to allow her to graduate in 3 years (she only needed 2 1/2 rears of additional credit).

    • anonymousskimmer says:

      synaxarion is arguing that elite schools these days don’t award admissions credit for (additional) APs (above a certain threshold).

      (parentheticals are my generous additions, which may or may not have been assumed by synaxarion)

    • anonymousskimmer says:

      Why is my above comment (April 19, 2019 at 8:09 am) no longer editable? Same non-editability with all but one of my other comments posted within the last hour.

  32. Riothamus says:

    There’s a concept in baseball called the replacement-level player. The idea is that everyone who has made it to the major leagues has gone through several rounds of being sorted into the best of the best. The consequence of this is that almost everyone has the same level of ability, because they are all from the far right of the Gaussian distribution…

    …and so they are trivial to replace.

    Turning to college, it looks a lot like people are investing a tremendous amount of time, stress, and money just to be disposable. If that’s the end state, there are much easier ways to get there.

    • anonymousskimmer says:

      Better to be a replaceable cog (today’s college graduates) than an unformed piece of wood (non-post-secondary). The repairman is going to grab from the piles of used cogs before carving a new piece of wood into a cog.

    • peak.singularity says:

      Huh, this is weird – my math teacher used to claim the opposite to justify why at my pretty good undergrad school, there seemed to be a much bigger relative skill difference between the average and the best student than what we were used to in high school…

      I had assumed that in a normal distribution, the skill level disparity between one person and the next one would be proportional to the height of the curve at that spot, so much lower in the center than on the edges !
      But statistics are a tricky business, I’ll have to think more about that… (I guess how exactly the selection is done matters a lot !)

    • Steve Sailer says:

      Being a replacement level major league baseball player (minimum salary $550,000) is a pretty good fate. Same with being a replacement level Harvard student.

    • eric23 says:

      Not all major league baseball players are replacement-level! Players have a spectrum of skill levels. Replacement-level is the far left of that spectrum, the point at which if you were any worse you’d no longer be in the major leagues at all, because they could easily find someone to replace you.

      Similarly, one could define such a thing as a “replacement-level Harvard student”. But most Harvard students would be more talented than that.

  33. kalimac says:

    What kind of extracurriculars was Peffern taking that were so cut-throat? When I think of high school extracurriculars, besides sports which I suppose are pretty competitive and always have been, I think of clubs and things like theatricals, which are what I did in high school, back in those noncompetitive days.

  34. AG says:

    What I don’t get is why admissions don’t give credit for working, and special credit for a recommendation letter from an employer, over extracurriculars. It’s down right discriminatory to fault someone for needing/choosing to make money over spending it on club dues, and it should also signal a level of work ethic that’s much harder to inflate than grades.

    • faoiseam says:

      What I don’t get is why admissions don’t give credit for working, and special credit for a recommendation letter from an employer, over extracurriculars.

      Two years ago, expensive college counsellors highly recommended that students get a manual labor job to show their credentials. Sadly, all the usual suspects immediately showed up with glowing recommendations from impeccably blue collar occupations. It is not difficult to get a builder to write a rec saying that you worked 8 hours a day on laying roofing all Summer. How can anyone check this?

      I suppose colleges could start demanding W2s to prove you were paid, but most small businesses are under such local control that even that would be effortless to game.

      • The Nybbler says:

        ROTFL. When I went to Maryland, a considerably number of my middle class classmates had spent summers doing roofing. Because it paid fairly well for manual labor. Too bad the Harvard thing didn’t exist then, I’m sure some of them could have worked something where the employers got the work done, and they got paid by both the wannabe Ivy students _and_ the employers to get the recommendation in the wannabe Ivy student’s name.

        I suppose colleges could start demanding W2s to prove you were paid, but most small businesses are under such local control that even that would be effortless to game.

        W-2s? From a roofer hiring temporary labor? Next you’ll be telling me they require I-9s too.

        • faoiseam says:

          W-2s? From a roofer hiring temporary labor? Next you’ll be telling me they require I-9s too.

          Let’s just say if I have a teenage US citizen, who doesn’t want to be paid or work, but would like an W2, then there are things that can get done.

      • AG says:

        It’s trivial enough to intersect the job information on the resume vs. their financial aid information. The point is not to ding students who can’t have extracurriculars (that Caltech description above sounded like an elitist nightmare to me) because they need to spend that “free time” making money.

        Having a job when you don’t strictly need one wouldn’t lend the same kind of credit. (Or rather, then it would be subject to the same kind of inspection as an extracurricular, i.e. you need to have tangible results of your own work that prove your ability, like a competition award or a project product.)

        • Edward Scizorhands says:

          Yeah, all me or my siblings got for our jobs while in high school was a reduction in our financial aid, since we could “afford” more.

  35. BBA says:

    Matt Yglesias, who can relate any discussion to restrictive zoning, thinks a reason why Harvard and MIT haven’t grown to keep up with demand for prestige is that the city of Cambridge won’t let them expand their campuses. I don’t know, I think even in the absence of political pressure the schools have plenty motivation to stay small and exclusive. Even when there is political pressure to expand, as in state university systems, a frequent result is to start new, less prestigious campuses and maintain the exclusivity of the older campuses – build UC Merced instead of expanding UC Berkeley.

    But then there’s the recent story of Montgomery County, Md., halting the issuance of new building permits for housing because its K-12 schools are at capacity and they can’t (or don’t want to) expand rapidly enough to meet demand. Um, what.

    • greenwoodjw says:

      Most of central MD land and education policy seems to be based on keeping Baltimore City contained and inflating housing costs.

    • Steve Sailer says:

      Harvard’s campus actually is relatively cramped, especially by the river, and is less impressive architecturally than you might expect. Expanding Harvard across the river has been going on for years, but it’s hugely expensive in the mode of Boston’s Big Dig. The local politicians and construction union bosses are generally not, shall we say, Harvard men and like to make Harvard’s Richie Riches pay through the nose.

      Caltech is similarly limited by a small campus.

      Some colleges, however, have huge amounts of room. For example, Stanford has an incredible 8,000 acre campus in the heart of Silicon Valley. (Stanford is currently leaning toward steady expansion of its undergrad ranks, perhaps 1% per year.) But there is very little pressure on institutions like Stanford to expand, except perhaps from rich donors who want to erect new buildings as monuments to themselves.

      A few colleges in Florida and Arizona have expanded enormously, but you almost never hear about them, except as punchlines in articles about the ongoing college admissions cheating scandal about places celebrities don’t want to send their children.

      • kalimac says:

        I live near Stanford, and can report that there’s strong pressure on Stanford from local governments not to expand too much. Mostly for traffic reasons – the big anti-growth concern around here – but also to preserve open space, of which Stanford has loads.

        I agree that Harvard’s campus is not very interesting architecturally. I was quite surprised when I visited there to see all the dull brick buildings. Yale is more interesting, and Princeton, with its fascinating large stone edifices, is absolutely beautiful.

        • I think the older Harvard houses, such as Lowell (which I was in), are patterned on the Oxford colleges.

          • kalimac says:

            Not physically, surely.

          • Yes physically, at least in appearance. I don’t know if things have changed since I went there, but the general layout of the older houses as I remember them is rather like what I saw in Oxford. I was never a student in Oxford, so don’t know how similar the interior is.

      • faoiseam says:

        Stanford is trying to expand, but the latest development is that the Palo Alto School system made a deal whereby Stanford would give the $120M. This annoyed Joe Simitian, the County Supervisor, as only he should be allowed negotiate, in his opinion. He fears that allowing other people to get kickbacks will infringe on his right to get kickbacks.

        The Palo Alto Online writes:

        the county would allow Stanford to move ahead with its expansion plan, which includes 2.275 million new square feet of academic space and 40,000 square feet of child care space and other support facilities.

        Stanford had also proposed including 2,600 student beds and 550 faculty and staff housing units in its plan, though the county indicated last month that it wants the university to provide at least 2,172 units of faculty and staff housing, along with 2,600 student beds.

  36. alexmennen says:

    Status update presentation had a slide outlining the results of a user study we did. We surveyed employers to find out what are the most vs least important details they look at when making a hiring decision.

    Out of something like 20 different options surveyed, “where the candidate went to college” was rated dead last in importance. “Formatting of resume” was rated as more important than “where they went to college” for making a hiring decision.

    This would seem to confirm conclusion #6 above

    This doesn’t address one of the possible ways that going to a prestigious college could help your career. The more prestigious a college you go to, the more of your college friends will end up being successful, and the more likely your social network is to help you get a good job. Employers don’t have to care whether you went to a good school for it to help you get a good job.

  37. Domineero says:

    “I failed in my original goal for this piece, which was to present an account of competition getting tougher across society. There wasn’t enough data,”

    If you want DATA about trends in university admissions and enrollment then I’m pretty sure the IPEDS database is what you’re looking for: https://nces.ed.gov/ipeds/use-the-data

    • faoiseam says:

      IPEDS is pretty useless for admission information as all the record is 25th and 75th SAT and ACT number of each exam type, total applications, number of admissions (full times and part time).

      That is a tiny part of what is used in admissions, so doesn’t really tell whether things are getting harder. What would be useful is GPA (weighted and unweighted), number of AOs and AP scores, per exam. Grades in core subjects. Math level (multi-variate calc, linear algebra, beyond etc.) Number of hour of volunteering, by decile. Number of leadership positions. State and National Awards per applicant, etc.

      The colleges do not give any data on the measures they actually use to admit people.

      • Domineero says:

        It depends on the questions you’re trying to answer. If you’re a teenager trying to minmax your schedule to get into a specific school, then yeah it isn’t very useful.

  38. mtl1882 says:

    It drives me crazy that we are still pushing the expensive college/grad school thing, often without any real awareness of what it will mean or what the student wants/is good at. It sounds sensible on its face to follow the “clear path,” but we’ve had enough time now to see that further consideration is needed for most people. Becoming an accountant is still practical and relatively secure—little else is. For all the mocking of degrees in literature or whatever, I feel like those are as useful as my political science and communication degrees. And accountants probably don’t need a full college education, nor do most other people—college education used to be considered almost entirely apart from professional training, something for the privileged and which cultivated connections and intellectuals. A simpler track should be available for most people who are not interested in this and who cannot afford it.

    I think the system will collapse somewhat—the expectations are so high that, while there are definitely people out there who can meet them, *so* many people are left out as to make it untenable. The payoff is so uncertain that in my opinion, while you probably need to get a college degree to have a good shot, I’d go somewhere cheap, do well enough, and get some good internships, and go from there. I did the traditional path, and I’m not nearly as bad off as most, but I have a lot of frustration with it on many levels. Mainly that so many people keep pushing expensive college and the expectation that it will get you somewhere, when that is clearly getting less and less true. As the comments show, you used to be able to be pretty uncommitted when you entered a field—now you have to be an expert. The biggest thing is the decline of apprenticeships. The system is selecting for a certain type of person who is certainly very capable and valuable, but other types are just as good in many positions, and they’re being culled early. You have to be aware of all this stuff at 14/15 years old, as many said, and a whole lot of that has to do with your parents, your connections, and in some cases, your own self-awareness and curiosity and desire to move in that direction, which is the thing that makes the most difference long term IMO. However, I think people with that level of self-possession often do well in many situations, and a lot of people who are hitting the checkboxes are doing so in accordance with expectations and targets not their own.

    I work in education, related to college admissions, so I think a lot about this. it is upsetting to see kids who are going to repeat my experience or worse, and whose parents tell them they are doomed if they don’t get into Harvard, but guaranteed an easy career path if they do.

    I was lucky enough to go to a good college with no debt, and a good law school with a little debt and a lot of merit aid. I turned down “better” colleges, at the college level due to fit and other things, and at the law school level mostly due to cost and location. I could have gone to an ivy, but I was at an age now where I appreciated the cost and was appalled at how much my parents had paid for four years of college. It was a city school, and one reason I chose it was because I could get a job or internship, and I had several valuable ones that helped me get into law school. My grades were very good in high school and college, but not valedictorian level or anything. And that was enough, with high test scores, strong essays, and my work/activity background. if I could do it over again, I’d have a much better perspective, and I kick myself for not having been more independent and aware of the options. At 18, I thought this was the only realistic, productive path, and it is embarrassing to me now. I’ve been able to find a lot of interesting jobs, but the one place I have no hope of getting into is a law firm, where grades are now just about everything, and only the top quarter of the class is taken seriously. I went to law school at the worst possible time, not realizing it. The education was a good one, though, so I can’t say it was a waste. But now I have a lawyer’s brain and I’m over analytical and going crazy in other jobs—even most law jobs are so much busywork that it would be an issue. I was fortunate to have some really good jobs/internships where I could analyze and write, and I miss it so much. Since then, I’ve found other uses for that skill set, but it is awkward because people always want to know why I’m not practicing or making more money.

    I went because I had been working for a lawyer for years during college, and it sounded good. But the killer curve meant I averaged about a B…and that was lethal. Before first semester grades were in, I got an awesome paid internship no problem. Once they were in, it was over, for the most part–same with making the journals. I was able to get two unpaid internships in the same semester, both of which were also great. All through this time I also worked. If I’d played my cards “right,” I would not have worked and interned so much, but that seemed to me more valuable, and I was used to it. I didn’t literally need money, but I didn’t want to live on soup, so I worked enough hours to get by, not full time or anything. From these positions, attorneys always offered to recommend me to a firm. They did not understand that the way they had waltzed in, admittedly mediocre students who had decided law school made sense like I did, was *not* an option. Firms like that will not accept a resume except through their system—most of which will reject below an A- automatically. I don’t practice but work with a lot of lawyers, who always wonder why. Then their kids go to law school…and they can’t get them a position unless it is a family firm. Of course, i could practice as a solo practitioner, and there are other options—all of which involve me working for a lot less money, with a lot more risk, and little training. You used to be able to apprentice and read law without going to school — I wish this was the case now. I could become an attorney starting in government or a very small firm/solo firm, but so far it just hasn’t been worth it to me to do it. I view it all as a bad investment, and I feel like all those years I was at school I could have been teaching myself these things, and working, while getting a degree at a less expensive college in a shorter period of time.

    Law firms are one of the few niche areas where this stuff actually matters—I agree that in most cases, it really is not nearly as big of a deal as people think. If I had realized this, I wouldn’t have gone to law school–it all came down to one exam per class, and I got about a B on every one but two. In those two, I had an A. In the classes that everyone seemed good at, I’d kind of zone out and follow the outline, and it was a false sense of security. It seemed easy. With everyone getting it, the curve would be harsh. I guess I just never got the rhythm right—but in any case, only 25% could end up in the 25% (something most people *cannot* understand–I deal with so many parents who expect 90th percentile for their kids like you can just aim for it and get there) In the other two classes, they deviated from prepared outlines, and the professors were somewhat eccentric—I got the top grades in both because I was able to pick up on this and more rigid thinkers were not—all a matter of the curve, again. I didn’t even necessarily do a better job in those classes—others just did a lot worse. One was law of corporations, and the professor had a stated Christian ethos—a lot of people didn’t like this and it drove them away. But it made things easier for me, even though I’m not religious, because when I had to make a recommendation, I would make one in line with Christian principles–in other exams, I couldn’t really see how to come down–not because I couldn’t make decisions, but because it would have been based on the desires and temperament of my client and what other avenues were closed. I had a hard time with relevancy, something I am normally good at–but based on my experience at that time, what was considered relevant on the exam differed widely–I really liked and valued the philosophical discussions, but on exams, it confused me. I feel like my past experience actually hurt me in many ways.

    I just feel like I’ve ended up spending hundreds of thousands of dollars to be unable to use my skills productively. It is wasteful. It’s not that I “deserve” better – it just seems like the whole process was helpful to no one. I should have gone in a different direction, and the sorting mechanism seems unhelpful. I’ve been successful in other areas, but I would have been more so had I not spent so much time in school and feeling hemmed in by my major. I’ve become a big believer in self-education and following a different path from the checkboxes unless you are a checkbox whiz—you will spend all your energy there, and if you aren’t that good at it, you should have spent it elsewhere. More options and flexibility, and apprenticeships, are necessary. Sure, most people don’t have the discipline to self-educate, but that is what apprenticeships are there for. The “structure” and thinking time provided by the current process is not paying off for most—I look around me and all I see is wasted talent. People being forced to work against their strengths, instead of being paired with someone who can take over their weak areas.

    Anyways, I could keep going on, like I have been. But I think the system has reached a real level of dysfunction and will have to adjust, or we’re going to have big problems. And I don’t mean that as in let’s give charity to those who can’t compete—I mean that much right now is form over substance–much of our talent is completely unexploited, and that hurts everyone, and people good at form are in the wrong positions. All as a generalization, of course. My experiences have been twisted — when I’ve worked harder and done a better job, it pays off at some point temporarily, and then manages to backfire, because they’re not selecting for substance in *most* jobs. There are definite exceptions. They don’t want someone finding something that conflicts with their plans—and, of course, I know that I can simply play along and not be the annoying one, and have done so—but then the thing I warned about rears its head, and I have to fix it, and the process repeats. Too much energy. Many of the greatest figures in American history were self-taught, and at the time this was not exceptional. I also agree that the common app makes more people apply widely, as do standardized exams. Each school used to have its own test—that is a very weird system in many ways, but it would cut down on this and allow schools to select for what they wanted. /rant

  39. gin-and-whiskey says:

    The people who are elite and “desirable” in non-academic ways (sports, legacies, donors, social connections, political activity, and affirmative action) are taking up a larger proportion of the class.

    Simultaneously, folks from other countries are upping their interest in high level academic admissions: Asia, India, and Europe. And they are doing a great job of it.

    Simultaneously, the increased connectivity and “smaller country” feel, together with the increased catering-to-students of modern colleges, probably have made travel easier and less stressful and have reduced the # of smart students who were artificially held back by unwillingness to travel.

    And of course the growing Asian population has set a much higher bar these days for what “smart” really means.

    The result is that for the folks who are applying BASED ON MERIT / INTELLIGENCE (as opposed to those other factors) it has gotten much much harder–it’s a multiplicative issue because it is those slots in particular which have simultaneously shrunk in number and also that area which has had a great increase in competitiveness.

    I’d say a 130 iq and appropriate school and SAT performance would have gotten you into a top 10 back in 1990. Now you probably need a 140-145, if that’s “all” that you’re going on.

    The problem (if you want to call it that) is that schools have move away from looking for “smart” people and now they look for people who may or may not be smart, but who have demonstrated “leadership” and such. The pool of spots for plain old smart people is dwindling fast, even at places like MIT. Those are the folks who are complaining.

    That said, if you are 125 iq progressive community organizer who writes poetry and casts protest pieces in bronze you probably have a better chance of getting into MIT than you would have had in 1990.

    • Douglas Knight says:

      taking up a larger proportion of the class

      larger than what? larger than in the past?
      No. Why do you believe that?

      It’s weird that you talk about MIT because it’s different in so many ways. If you mainly know about MIT, you’re wrong to extrapolate to other schools. At least MIT has room to move in this direction, but other schools don’t. (I was told that Princeton has an explicit plan to move in the opposite direction.) But I doubt that MIT is moving in this direction. What about Senior House?

  40. Icedcoffee says:

    It all makes sense now: Thanos didn’t get into Space Harvard!

    Maybe prestige just can’t keep up with population growth. Or maybe its because so much of our economic growth is centered on tech, which employs comparatively fewer people than older prestige jobs like Big Law or Big Consulting.

  41. jddt says:

    @Scott:

    You asked in your original post about changes made in 1992.

    So the stories I have been told by my elders and betters sounds like this:

    Not all that long ago, higher education in the UK was highly heterogenous: with apprenticeships, specialist colleges (for example, nursing schools attached to hospitals, art colleges, electronics colleges and so on…). Due to, as far I can tell, class snobbery, universities were always seen as the greatest form of educational device, even though they may be the worst way to train people in any particular thing — anyways — the education system seems to have charted a course to converge all higher education to the university model.

    This started with specialist colleges and apprenticeships, family businesses, or on-job training, and so on, being replaced with technical colleges with trained you in a variety of technical things (though perhaps not as well as on-job training, and at lower productivity and so on — I have no evidence for either of these claims), and then collocated ones were merged into Polytechics, and then these things in 1992 were turned into universities — which usually gobbled up the non-technical colleges, such as arts colleges, nursing schools, teacher training colleges, and so on.

    Now, the change from training colleges to universities was quite deep and people have been despairing ever since: because these institutions have gone from being about training people to new skills, to be about getting people in the door, so they can get public funding, to perform research — since in the UK system this is formally what universities are for. Also, the teaching is generally now in the university style of a chap standing at the front of a room speaking at sleeping people who learn very little before disappearing. For example, my friends in nursing tell me the quality of qualified nurses have dropped dramatically now that nurse training is about writing essays and sitting in lectures rather than treating patients in a hospital: particularly with regards to stress and care-giving — you can be quite well invested in your degree by the time you have to care for a patient, and find you aren’t actually into that sort of thing — so you don’t leave nursing (which makes both you and nursing better off) but you instead stick at it because this is your career now and you’ve already taken out massive loans — whereas in the old system you’d find this out much earlier. Also, the people teaching you how to be a nurse have to engage in research of some sort — because they are lecturers in a university — one of my friends said that her lecturer fulfilled this requirement by ever-expanding the Bristol Poo Chart to ever more types and varieties, to the interest of no one. I went to a panel discussion recently where lecturers at what used to be an Art College but is now part of the University, were talking about how in the old days they were paid to be teachers, and doing their own art with the students was part of that — giving them both fulfillment but also basically they were in a system which made sense — but now they have to submit applications to research to get funding to do what they were doing before — and traded tips on the best ways to make what they’ve been doing all along sound like original research.

    I have to imagine that things should have been the other way around: we’ve have better doctors if they were trained more like nurses, on the job; but now we have nurses who are trained like doctors and now everything is terrible.